Free Energy | searching for free energy and discussing free energy

New theories about free energy systems => Theory of overunity and free energy => Topic started by: Zetetic on April 14, 2015, 10:59:57 PM

Title: A possible violation of the Law of Conservation of Energy
Post by: Zetetic on April 14, 2015, 10:59:57 PM
Hello.

I thought you guys might be interested in this.

It seems to me that you guys have two hurdles to overcome; one theoretical and the other practical.  To successfully build a free energy device you must, one, find an exception to the Law of Conservation of Energy and, two, then get a device mechanically to work with this exception.

I believe I have found a flaw in the Law of Conservation of Energy.  Unfortunately, I don’t think this particular flaw I found can lead to a mechanical free energy device.  But, having said that, if I’m right and I found an exception to this law, and thus cleared the theoretical hurdle, I thought you guys still might be interested in this.

I posted my argument at Science Forums.  The original post of my argument was really long and I don’t think anyone really read it fully.  The comments that followed were all tangential and didn’t address the issue I raised.

I kept thinking about the question and was able to get it down to two pages.  I posted this new formulation of the question in the same thread on April 10th.  There were no responses.  But I know they were reading it based on the “view counter” increasing.  I tried bating them into commenting (e.g. “Silence, generally, means consent”).  And finally today, April 14th, I got one response.  But it was meaningless.  The comment was not that there was a logical or factual flaw in my question but rather that had a “poor use of fonts.”  And this, to me, seems like another concession that they can’t find an actual flaw in my argument.

So, anyway, I thought you guys might be interested in this.  Here is the link to post #21 where I propose the shorted and sharper argument:

http://www.scienceforums.net/topic/87489-a-magnet-is-demagnetized/page-2

If any of you have ever posted anything on Science Forums you know that they love to make snarky comments putting people like you and me in our place when we question the “known Laws of Physics.”  Their silence is, I believe, very telling.  It makes me think that I may really be onto something.  I may be wrong (I am all the time), but right now I think I really may have found something.

Again, I don’t think it leads the mechanics needed to make an actual free energy device.  But, maybe it does and one of you can find it.  I don’t know.

If you do go and read this discussion at Science Forums and if you then choose to comment, please be careful.  If I’m right and I’ve found a violation of one of their most fundamental beliefs, and that’s why they can’t respond, I’m sure they would love to lock up the thread and end my blasphemy.  I’ve been very careful to try to formulate everything in the form of a question and to try to avoid anything that would violate their rules.  All they need is for me or someone else to make a declarative statement like “physics is flawed” or “this proves the Law of Conservation of Energy is wrong” to lock the thread up and move it off to the dustbin of their “Speculations” folder.

So, it’s not a free energy device, but I thought it might be related enough to the stuff you guys are into that you might be interested.

And that’s it.  (Again, I may be wrong but right now I think I really may have found something.)

Take care.


Title: Re: A possible violation of the Law of Conservation of Energy
Post by: ayeaye on April 16, 2015, 05:36:19 PM
I'm likely not into all the scientific subtleties of this, and what laws of physics it may or may not violate. Going into equations would take aeons there i guess, so i prefer simple principles. Thus sorry if my question is somewhat naive. So what are you going to do, achieve a continuous movement by demagnetizing a permanent magnet using heat? This may well be possible, but then does not heating the magnet take energy?

There may be overunity in permanent magnets, but by my experiments i found that this is not enough for continuous rotation. Overunity is not the same as permanent motion. But can you say where the energy on the image below comes from? I don't say that the law of conservation of energy is violated, i just don't know where the energy comes from. And for making  free energy devices, i also don't see that it is important to answer to that question. I did the experiment of that, see this video https://archive.org/details/Flcm3 (https://archive.org/details/Flcm3) . This experiment was done just so that one can repeat it, and see oneself whether the results are the same. Just make sure to use magnets of the same size and strength, and a disc with a good ball bearings.

The result was that when starting to move from almost the same point, the disc moved some two times more by the chain of magnets, than in the opposite direction. This should not be so when all the energy came from moving the disc to the starting position, and all the movement was only caused by repulsion of the first magnet. This does not provide a continuous rotation with permanent magnets or electromagnets, so for that it is useless. Which does not mean that such effect may not cause overunity when charged particles move in a solid state device.

Thank you for your attention.
Title: Re: A possible violation of the Law of Conservation of Energy
Post by: Zetetic on April 16, 2015, 09:40:30 PM

hey ayeaye ...

Thanks for the video link.  Cool design!

I watched the video you linked to (Field lines chain motor 3/4), as well as the links to the previous video (Field lines chain motor 1/4) and the next video (Field lines chain motor 4/4).

(Is there also a “Field lines chain motor 2/4” and I just missed it?)

The “Field lines chain motor 1/4” worked as I would have guess it would.  Whether you have one magnet that is capable of moving or several aligned magnets capable of moving, I would expect them to move so that the pole of the moveable magnet (or magnets) moves away from where it is repulsed by the external and fixed magnet and to where it is attracted to the external and fixed magnet (the one in your hand).

I was surprised by the results of “Field lines chain motor 3/4.”  I did not expect the movable magnets to spin past, and to spin so far past, the end of the chain where the pole of the moveable magnets is attracted to the external and fixed in place magnet.  That was interesting.

What my guess is that you have found a magnetic arrangement, in “Field lines chain motor 3/4”, where the repulsed starting point is a greater force than the attractive point at the end of the row (curved row) of magnets.  And so, even though the row of magnets are attracted to the external fixed magnet (as was seen in “Field lines chain motor 1/4”) in this arrangement the force and the resultant velocity from the repulsed starting point is enough to break free and move beyond the greatest point of attraction between the moveable magnets and the external fixed magnet in “Field lines chain motor 3/4”.  So, I was surprised by the results, but it makes sense that such a configuration as you found could be made.

“Field lines chain motor 4/4” did work as I would have expected.  By going from a row of magnets with a south end and a north end (“Field lines chain motor 3/4”) to a full circle (“Field lines chain motor 4/4”) means going from a system where there is a point of greater repulsion and a point of greater attraction to a system where there is no point of greater repulsion and no point of greater attraction (assuming a perfect physical system with no imperfections).  And what I would expect in this case is, after the moveable magnets have been set in motion (by your hand), for them to not continue in motion by a force from the arrangement (since there is no greater point of repulsion for them to get away from and no greater point of attraction for them to move towards), but rather move for a while due to having been set in motion by your hand and then eventually come to a stop due to friction (your device, like all real world devices, is not frictionless and friction brings motion to a stop (while generating thermal energy).)  And the fact that your device in “Field lines chain motor 4/4” tended to come to a stop at the same point in the circle of magnets also is what I would have expected.  In a hypothetical perfect circle of magnets I would expect it to come to a stop at different random points.  However, in a real world device (where there will always been some slight differences, imperfections, in the manufactured magnets and some slight differences (even if only very very tiny) in the size and shape of the gaps between the magnets) it is expected that there would be some point around the circle of magnets where the attraction of the circle of magnets is greater (even if only slightly greater) with the external fixed magnet.

I know the results of “Field lines chain motor 3/4,” where the curved row of magnets moved well beyond the external fixed magnet, might seem like this would lead to continuous motion forever when then moving from this configuration to a complete circle of magnets, but I think that by going from a row to a complete circle you are eliminating the dynamic (a point of greater repulsion and a point of greater attraction) that caused the movement in “Field lines chain motor 3/4.”

I think that if you were able to physically build a truly perfect “Field lines chain motor 4/4” (with no slight imperfections or variations) that it would come to a stop, due to friction, at random points, while if you were to build such a design (with real world materials and real world placements that will always be something less, even if only slightly less, that perfect) then you’d see it come to a stop in one or two (or so) places more often than not.

But ... this is just my opinion.

---

If you’re interested in the Law of Conservation of Energy and how it relates to free energy devices, I can explain it to you in a couple of very short and common sense language paragraphs (no Physics jargon needed, no math needed, and I promise it will only be a small fraction of the length of my review of you design).

So, if you’re up for that, brief, discussion/explanation ... let me know.

---

Again, cool design!  Thank you for showing it to me.  I hope my analysis of it is of some use.

Take care!

- Zet 



(PS:  While I believe I understood what you were going for in the videos, I’m sorry, but I don’t think I quite understood what you were trying to explain with your drawing.)



Title: Re: A possible violation of the Law of Conservation of Energy
Post by: ayeaye on April 16, 2015, 11:06:54 PM
What i was trying to explain with my drawing is this. A north pole of another magnet is moving through the field of the magnet, which is as it is drawn there. And it moves in the direction shown by the arrow. Now when this pole of the magnet is where the field lines are horizontal, then it moves by the field lines, this is what field lines mean. And when moving by the field lines, it gets a propulsion, to the direction where these field lines go.

The only thing which may work against it, is a repulsion, that is going through the field lines when their direction is opposite to the movement. But as can be seen on the drawing, this never happens, because in the worst case the field lines are perpendicular to the movement, and never opposite to the movement. Which means that the moving magnet gets extra speed when moving through the magnetic field of the other magnet, and thus an additional energy. This should not happen when there is no overunity, the pole of the magnet should go through the field of another magnet without getting any additional energy. Thus the additional energy should come from somewhere, the question is from where.

The reason why this happens is that magnetic field is not symmetric. It were symmetric if it were just radial lines coming from the magnet, like a field around a sphere. But it is not like that, and thus magnetic field is not symmetric. And every asymmetric field can in theory do work.
Title: Re: A possible violation of the Law of Conservation of Energy
Post by: Zetetic on April 17, 2015, 01:28:40 AM



ayeaye,


I guess I’m not sure how you have the two magnets oriented towards one another in the drawing.

In your videos I thought your magnets (the set moving magnets (row and full circle) and the fixed magnet) were orientated relative one another as they are in A in the drawing below.  (But, of course, in your videos it’s the other magnet or magnets that is in motion and the other magnet that is fixed.)

And so, this is how I thought they were also oriented in your drawing.

But, now, based on your last response, I think maybe in your drawing you have the orientation different, as shown in B in the drawing below.

I hope to understand what you’re getting at, but we need to clear up my understanding of your point.

A?
B?
Or, something else?


- Zet



Title: Re: A possible violation of the Law of Conservation of Energy
Post by: ayeaye on April 17, 2015, 02:18:49 AM
It is always A. In my experiment, the upper magnet in A is the stator magnet, and the lower magnet in A is one of the rotor magnets. The stator magnet is a kind of long, so only one pole, the north pole, really matters, the effect of the other pole is very small and can be disregarded. On my drawing, the upper magnet in A is shown as moving, and the lower magnet is shown as standing still.

I used a long cylindrical neodymium magnet as the stator magnet. It is important in that experiment that the stator magnet is strong, and the rotor magnets are small. Mainly because the rotor is lighter then.

So the upper magnet is really the stator magnet, but it is a kind of opposite on the drawing, i showed it so because i thought that it's better to explain it that way. As it doesn't really matter which one of the magnets stands still and which one moves, because it only matters that the magnets move relative to each other.
Title: Re: A possible violation of the Law of Conservation of Energy
Post by: Zetetic on April 17, 2015, 03:41:49 AM


ayeaye,



Okay.  Your magnets are always orientated like in drawing A in reply #4.  That makes sense.  Thank you for clearing that up for me.

I now have a second basic question about your drawing.  Hopefully we can also clear that up for me and then I can finally get to understanding and then addressing the point you are making with your drawing.

In your drawing you show the moving magnet before reaching the fixed magnet and then you show the moving magnet after passing the fixed magnet.  And, when the magnets are in those positons, you indicate that there is “no repulsion.”

I think I get your point, but I think the logic needs to be fleshed out a little bit more.

When the magnets are in position X (in the drawing below) there is no repulsion (so I agree with your drawing about that), but there is attraction (which is not indicated in your drawing).  And this attraction inhibits the leftward movement of the moving magnet.

And when the magnets are in position Y (in the drawing below) there is no “repulsion” in the sense of pushing the magnet rightward.  However, when the magnets are in position Y there is both attraction pulling the magnet leftward and (I believe) repulsion pushing the magnet leftward.

This is counterintuitive.  It seems as if when the moving magnet is to the right of the fixed magnet that the moving magnet should be repulsed to the right.  However, the magnetic lines of flux, obviously, extend out beyond the physical location of actual magnet itself.  And so, when the moving magnet is just slightly to the right of the fixed magnet there are repulsive magnetic lines of flux to the right of the moving magnet and this will push the moving magnet leftward.  It counterintuitive, but if you think about it it makes sense, and it’s one of those weird aspects of magnetic interaction that I find fascinating.

But if the magnets are in position Z (in the drawing below), if the moving magnet is just slightly more to the right of the fixed magnet, then there is repulsion which will push the moving magnet rightward (and this is not represented in your drawing).

This can be seen in your video “Field lines chain motor 1/4” at about 1 minute and 21 seconds.  You place the fixed magnet at the start slightly more to the right than in the other iterations and the moving magnets (in this case it is the other magnets that are the moveable) are repulsed away.

So, again, I’m trying to understand the basics of your drawing before trying to understand the point you are making with it.

There is no repulsion at point X (as you indicate in your drawing), but there is also attraction (which is not included in your drawing).

There is no “repulsion” at point Y (as you indicate in your drawing) in the sense of pushing the moving magnet rightwards (but there is a leftward repulsion).

And there is repulsion when the moving magnet is to the right of the fixed magnet, point Z, but this occurs when the moving magnet is slightly more to the right of the fixed magnet than point Y (and this is not indicated in your drawing).

No?

Did I get something wrong?

Let me know!

Again, it’s a cool design.  Thanks for sharing it!


- Zet

(PS:  I agree with you that it doesn’t matter which is the fixed magnet and which is the moving magnet (a difference between your videos and your drawing) but only relative the orientation between the two matters.)


Title: Re: A possible violation of the Law of Conservation of Energy
Post by: ayeaye on April 17, 2015, 05:23:25 AM
The pole of the magnet moves by field lines. What concerns the repulsion in the beginning and the attraction in the end, then this happens only when the field lines go from a pole of the standing (lower) magnet to the pole of the moving magnet. This is insignificant when the lower magnet is a part of a chain of magnets. Then the field lines from its south pole go to the north pole of the previous magnet, and the field lines from its north pole go to the south pole of the next magnet.

This drawing was mostly drawn with the field lines chain design in mind. With a stand alone magnet some field lines certainly go to the pole of the moving (upper) magnet. Then it may just depend on how long are the field lines. Whether there is any overunity in that case, i don't know, this drawing was to explain the results of the field lines chain experiment.
Title: Re: A possible violation of the Law of Conservation of Energy
Post by: ayeaye on April 17, 2015, 06:03:47 PM
So the concept basically is, that the magnetic field is an asymmetric field, so in some circumstances it can do work. And my experiments seem to confirm this. Yet it cannot provide permanent motion at least in a mechanical way.

OK, tell about your thing, what is the reason why it can be overunity?
Title: Re: A possible violation of the Law of Conservation of Energy
Post by: Zetetic on April 17, 2015, 07:56:25 PM


I thought I was getting a better handle on your proposal, but now I’m not so sure.  I feel like maybe I’ve taken one step forward and two steps back (in trying to understand it).

I’m trying!

Based on what you wrote in Reply #7 I have yet another basic question.

There is a concept in the physics of magnetics called the “unmagnetized” state.  “Unmagnetized” is different from “demagnetized.”  Here is a link to a description of the former (if you’re interested):

http://www.irm.umn.edu/hg2m/hg2m_d/hg2m_d.html

It’s somewhat technical, but I think it may be relevant to what you are working on.

When a magnet is “demagnetized” the poles of the particles within it become randomized and so there is no longer an external magnetic field and thus the effects of the field are gone.  And when a magnet becomes “unmagnetized” (as a ferromagnet has the natural tendency to do) the magnet moves to a state with more and more multiple poles within the single body, and this reigns in the external magnetic field and so here also the effects of the external field are also (essentially) gone.

I’ve never heard of someone making a ring of magnets in order to reign in the external magnetic field.  But in your Reply #7 it seems like you might be suggesting that that is what you are doing.  And, again while I’ve never heard of someone doing this, it make sense that your ring of magnets would essentially lead to a state of “unmagnetization” (in the sense of the word used in the link above).

But, if this is what you are suggesting, then I’m confused.  This would then mean the “unmagnetized” ring would not significantly interact with the external fixed in place magnet.  And isn’t the interaction between the ring of magnets and the external fixed magnet the whole point of your design?

Again, I’m sorry.  I’m trying!



---



Okay, back to the idea in my original post.  Thanks for asking!

The link in the original post does not lead to an argument for an Overunity Device but rather to an argument for why the Law of Conservation of Energy is false.  And the two things, while different, are related.  And I can explain this in very simple terms.

The Law of Conservation of Energy says that while energy can change forms the total amount of energy remains the same.

There are different kinds of energy.  Motion is a form of energy (“kinetic energy”).  Heat is a form of energy (“thermal energy”).  A ball some distance above the surface of the Earth is a form of energy (“potential energy”).  And so on.

This “Law” says that energy can change from one form to another (e.g. motion can become heat and heat can become motion) but the amount of energy remains the same.

So, for example, say I roll a ball across the floor.  When the ball leaves my hand it is in motion, and this motion is a certain amount of energy, and let’s say its 10 units of kinetic energy.  As the ball rolls across the floor it eventually comes to a stop.  The 10 units of kinetic energy is now gone.  What brings the ball to a stop is the friction between the ball and the floor.  And friction generates heat.  And so, when the ball comes to a stop, there is a loss of 10 units of kinetic energy but there is also an equal increase of 10 units of thermal energy (heat).

And so, in the ball rolling across the floor example, energy has changed forms (motion becomes heat) but the total amount of energy remains the same (10 units of kinetic energy becomes 10 units of thermal energy).



Okay, why is this relevant to OU?

Say someone was successful at making a working device that once set in motion continued moving forever (such as if you were able to get “Field lines chain motor” to spin forever).  There is a certain amount of energy put into the system at the start.  It takes energy for your hand to originally set it into motion (there is a decrease in biological energy and an equal increase in kinetic energy in the form of the spinning disk).  Now, say, it keeps moving.  This would mean there is no decrease in kinetic energy.  But, as it moves there is friction (between the spinning disk and the bearings and the axel and between the spinning disk and the surrounding air and so on).  And friction generates heat (thermal energy).

So, if your design (or anyone else’s in this forum) was to keep moving after being set into motion, despite encountering friction, there would be no decrease in kinetic energy while all the time it is moving more and more thermal energy would be generated.  Energy would be created!  The total amount of energy within the Universe would increase!

And this would be a violation of the Law of Conservation of Energy.  The total amount of energy would not remain the same.  There would be no decrease in kinetic energy while there is an increase thermal energy.

(There is also the quest for a perpetual motion machine of the second kind which does not violate the Law of Conservation of Energy but rather violates the Second Law of Thermodynamics, but that’s a different topic.)



So, back to my original post (and the argument it links to).

I’ve spent years playing around with trying to build a perpetual motion machine (free energy device, overunity device).  But I’ve also spent years just playing around with the logic of the Law of Conservation of Energy itself.

And I believe I’ve been able to craft an argument that shows a violation of the Law of Conservation of Energy, which means, if I’m right, shows the Law of Conservation of Energy is false.

Unfortunately, the specific violation (and thus disproof) of this “Law” that I found does not also (I don’t think) lead to the mechanics needed to design and build an OU device.

But, if I’m right, and I have shown this “Law” to be false, this would validate the efforts of you and me and everyone else in this forum.  If you were to show your “Field lines chain motor” to an actual physicist, he or she wouldn’t waste any time looking at the mechanics of it to see if you’ve got something, they would “know” that there must be a flaw in there somewhere because the Law of Conservation of Energy tells them that such a device as yours could never work and so they’d just dismiss it out of hand without any examination.

If I’m right and I’ve found a flaw in (and thus disproved) the Law of Conservation of Energy in my argument (linked to in the OP), and if this could become established and well known, then efforts like yours in your design would be given credibility in the “established” scientific community.

And that’s it.

That’s the point of my post here.

If you read my argument in the link in the original post, let me know what you think!

(However, if you want to talk more about the mechanics of your design I’m happy to do so ... I would enjoy doing so!)



- Zet


(PS:  Oops.  I promised in my Reply #2 to explain the Law of Conservation of Energy and its relation to OU in a couple of short paragraphs.  Oops.  I guess I went a little bit overboard here.  Sorry.)


Title: Re: A possible violation of the Law of Conservation of Energy
Post by: ayeaye on April 17, 2015, 10:34:01 PM
I just did experiments, to see what happens. Yes i also tried to make a full circle of magnets but this, maybe indeed became what you say "unmagnetized". That is, the poles of some magnets shifted, for some unknown reason. And this is why the continuous rotation was not possible.

The experiments showed overunity only with shorter chains of magnets, something changed whenever it was made to full circle.

My hand, all you have to do is to move the disc near the top of the peak near the first magnet. You feel it, and you have to move it as close to that peak as possible. Then it moves in different directions depending on to what side of the peak you moved it. Your hand of course provides energy, but if you move it near the peak every time, the energy you provided should be the same both when the disc moves in one direction, or in the direction opposite to it. Now what shows overunity is that it moves some two times more in one direction, than in the opposite direction, in spite that the initial energy you provided was the same.

I did it with my hand, you can try it with your hand, and if the magnets and the disc are well repeated, you should get the same results. I know that its better to use mechanical things, and not hand, as hand is not very reliable, and the video cannot show all the slight movements of the hand, which could affect the result. But the experiment is simpler and thus much more easily repeatable that way. Because what i also know is that no experiment can be provable, experiments can only be repeatable. Thus easy repeatability is better than more provability. So i made the experiment as simple and as easily repeatable as possible.

I used a computer's processor fan for bearings, but i oiled it well. And i used only mounting tape for fixing things. Tesa, scotch, 3m, whatever you get works well. But magnets are a bigger problem, i don't know where to get so small ceramic magnets which i used in the video 3/4. I got them from some small decorations which they put on the refrigerator door, but not all of them have so small magnets. One may try very small neodymium cube magnets, which can be bought, but i'm not sure whether the result will be the same.

What concerns the law of conservation of energy, then i think it is in the very essence just an expression of the balance in the universe. So even if you can show that it is not valid in one process, it is valid more generally. But no matter what the theories, conservation of energy or the energy came from some unknown source, i don't want to speculate where the energy came from. Because if i did, and if it happens that my guess was wrong, then for that same reason it would be considered that all that i said was wrong.
Title: Re: A possible violation of the Law of Conservation of Energy
Post by: Zetetic on April 18, 2015, 04:36:36 AM

ayeaye,



Thank you for continuing the conversation on Conservation of Energy.

“So even if you can show that it is not valid in one process, it is valid more generally.”    -ayeaye

This is the crux of the issue.

The Law of Conservation of Energy is a “Law” and not a “Principle.”

Mainstream Physics does not claim that energy is “generally” conserved.  It claims that energy is always conserved.  They claim that no one can ever “show that it is not valid in one process.”

If they are right, then all the attempts at creating an Overunity Device as seen in this forum are nothing more than a waste of time.  A successful Overunity Device would be a mechanical device that violates the Law of Conservation of Energy, and if they are right, then no Overunity Device attempt will ever succeed.

This is the issue.  And this is the point of me posting a link to my disproof of this “Law” in this forum.

And so it’s my hope and you, or anyone else in this forum who disagrees with mainstream Physics and believes an Overunity Device is possible (and therefore necessarily believes the “Law of Conservation of Energy” is not a “Law” but a “Principle”), might then discuss with me whether or not my argument (linked to in the OP) does actually disprove this “Law.”

That’s it.  That’s my purpose in posting here.


---


I think I’ve figured out why you get the results you get in “Field lines chain motor 3/4.”


Think of a hypothetical frictionless pendulum (see Q in the drawing below).

The pendulum starts out at rest and at a certain height (Q.1).  It then swings down and increases in velocity until it reaches its lowest point (Q.2).  And then it swings back up and decelerates until it finally comes to a stop and at the same height from which it started (Q.3).

The velocity reached at its lowest point is the exact amount of speed needed for it to then reach the same height on the other side (again, in a hypothetical frictionless world).

The same thing can be done with a magnet on a rotating disk and an external fixed magnet (see R in the drawing below).  Again, this is in a hypothetically frictionless world.  The two magnets (the moveable one and the fixed one) are orientated so that their dissimilar poles, which are attracted to one another, will pass by one another facing one another.

The moveable magnet starts at a rest at a certain distance from the fixed magnet (R.1).  It then rotates towards the fixed magnet and increases in velocity until it reaches the fixed magnet (R.2).  And then it continues to rotate beyond the fixed magnet and decelerates until it finally comes to a stop at the same distance from the fixed magnet from which it started (R.3).

The velocity reached when it is at the point of the fixed magnet is the exact amount of speed needed for it to then reach the same distance from the fixed magnet on the other side (in a hypothetical frictionless world).

Now the same thing is done again, but with the magnets orientated relative to one another as they are in “Field lines chain motor 3/4.”

The moveable magnet starts out at rest just to the left of the fixed magnet where it will be repulsed to the right and not to the left (S.1).  But at this point, and this is the key point, the moveable magnet is both repulsed and attracted to the fixed magnet.  In this position, the right end North Pole of the moveable magnet is repulsed by the North Pole of the fixed magnet and the left end South Pole of the moveable magnet is attracted to the North Pole of the fixed magnet.  The moveable magnet then rotates to the left and increases in velocity until it reaches the point of greatest attraction between the moveable and fixed magnets (S.2).  It increases in velocity due to both repulsion and attraction.  It then continues to rotate beyond the point of greatest attraction and decelerates due to the attraction between the moveable magnet and the fixed magnet.  It decelerates, and again this is part of the key point, due now only to attraction (S.3).

The velocity it reaches when at the point of greatest attraction is from repulsion and attraction and so will be greater than the velocity needed to escape and move beyond the attraction (and only the attraction) while it is decelerating on the other side.

This is why, I think, you get the results you do in 3/4.  (And, even though you’ve taken steps to reduce the friction as much as possible, in your real world model (as in all real world models) there is friction and this is my guess why it then finally does come to a stop after escaping the attraction with the fixed magnet.)

So, you and I may disagree about the reason why for the results you get.  I hope I haven’t offended you with an alternative theory.  Maybe I’m wrong.  But this is what makes sense to me right now as I look at it.  I’m happy to continue to discuss it with you and even to try to find flaws in my analysis!  (I’m interested in truth in the end and I’m not interested in “winning” arguments in the passing.)

Again, I hope you don’t mind me offering an alternative theory.

Take care!

- Zet




(PS:  I’ve never built your exact model in “3/4” but I’ve built similar kinds of models.  I, too, have “felt” the “peak” of the magnetic repulsion.  I’ve always found this a fun and fascinating experience!)




Title: Re: A possible violation of the Law of Conservation of Energy
Post by: ayeaye on April 18, 2015, 06:25:18 AM
My drawing was about one magnet in a chain of magnets, and i have only seen overunity with a chain of magnets. Ultimately the friction stops the movement of course. But overunity is because the repulsion of the first magnet of the chain should be the greatest propulsion there, if the other magnets in the chain do not add additional energy. But because the disc moves some two times more by the chain, they evidently do, the way the drawing shows.

The energy necessary to move the disc to the peak of the magnetic force at the first magnet in the chain, is also equal to the energy released by the repulsion of that magnet. When the disc moves some two times more than that by the chain of magnets, the energy released is more than the input energy. The known input energy that is, because the only known input energy is the energy necessary for moving the disc to the peak of the magnetic force.
Title: Re: A possible violation of the Law of Conservation of Energy
Post by: allcanadian on April 18, 2015, 08:33:37 AM
@Zetetic
Quote
If any of you have ever posted anything on Science Forums you know that they love to make snarky comments putting people like you and me in our place
when we question the “known Laws of Physics.” Their silence is, I believe, very telling.  It makes me think that I may really be onto something.  I may be wrong (I am all the time), but right now I think I really may have found something.
Again, I don’t think it leads the mechanics needed to make an actual free energy device.  But, maybe it does and one of you can find it.  I don’t know.
You make a very good case and I had to think on it for a while. I believe the conservation of energy relates to the known laws of physics however one can never know all of the near infinite number of ways the law(s) may be applied.
You know within a few minutes of reading your post I may have found a way to prove the paradox. Now imagine we had a very strong permanent magnet insulated from a hot metal plate in a box by a near perfect insulator such as aerogel. If we had metal balls inside the box then they would be attracted upward to the magnet however they would first impact the metal plate generating heat and this heat would partially demagnetize them causing them to fall to the bottom and repeat the cycle. Obviously the metal plate must get hotter from these impacts regardless of whether it was preheated to start the process or not. As well the impact in itself is also know to disturb the magnetic domains only increasing the effect. We could also pull a vacuum in the box to reduce the heat loss from top to bottom. Now if the bottom of the box dissipated part of the heat generated by the impacts cooling the balls just below the Curie temperature then we would theoretically have Maxwells demon.
 
The metal balls would in effect be the Demon generating heat through the transformation of kinetic energy to thermal energy on impact which in effect maintains their motion. However we should note the transformation is simply the macroscopic kinetic energy reduced to kinetic energy acting on a smaller scale we call heat. This also relates to a statment by Faraday-- "It does not matter how the change occurs only that it does". At which point we might ask would this be overunity? Well no because the universal radiation field induced the charge on the fundamental particles in matter which caused them to move which created a displacement field we call magnetism which was then disturbed by the thermal energy created on impact. The supposed extra energy would still be real energy transferred from the external field to make up the difference it's just that we never thought of that. Overunity is simply a convienient term to describe the fact we don't understand what the hell is going on, lol.
Would this ill concieved conservation breaking demon machine work in reality?, most likely not however is does raise some interesting questions.
AC
 
 
Title: Re: A possible violation of the Law of Conservation of Energy
Post by: Zetetic on April 18, 2015, 09:43:17 PM


allcanadian,



Thank you for reading my argument against (hopefully my “disproof of”) the Law of Conservation of Energy.

-

“I believe the conservation of energy relates to the known laws of physics however one can never know all of the near infinite number of ways the law(s) may be applied.” – allcanadian

Are you saying that the Law of Conservation of Energy is generally true but not absolutely and always true (as ayeaye seemed to suggest above)?  If so, I agree with both of you guys.  But this means we are all in disagreement with mainstream physics.  And that’s the point of my argument in Science Forums.

My argument (linked to in the OP) is a very simple piece of logic that (hopefully) shows that there is a flaw in the logic of the Law of Conservation of Energy.  And this means we can know that this “Law” is false.  And we can know that it is false to a logical certainty!  (No experiments needed.)

And this, I would think, would be something of interest to people who are trying to build mechanical devices where more energy is outputted than is imputed (which, if successful, would be a violation of the Law of Conservation of Energy, and where, alternatively, if this “Law” is in fact true, then these devices could never really work).

That’s it.

(If anyone does read my argument linked to in the OP, please feel free to just read the question and ignore or skim over the endnotes.  The question is very straight forward and simple, but the endnotes, at times, are a bit tedious.  The endnotes are there simply in anticipation of the likely issues to be raised in response to my argument.  They are superfluous.)

-

Before I comment on your two balls in a box example, I want to make sure I understand the set up.  In drawing M below I have visually described what I think you are saying.  There’s a good chance I’m wrong, so please correct me if so.  That’s the point of me drawing it out: to make sure I understand your set up.  And then, once we are on the same page, I’d love to comment on it and discuss it with you.

However, please note (as I’m sure you know) that Maxwell’s Demon is a Second Law of Thermodynamics issue and not a Law of Conservation of Energy issue.  Both “Laws,” and the hopeful disproof of both “Laws,” are relevant to people like us who want to see successful perpetual motion machines.  But they are two different and distinct theories, and the mechanical devices that purport to violate them are two different and distinct kinds of mechanical devices.

If my argument linked to in the OP tends to inspire a Second Law of Thermodynamics analysis then, my bad, it should not have.  There is a “heat pack” in that argument.  But it is only there for the purpose of demagnetizing (raising above the Curie temperature) the magnets, and therefore ending the mutual attraction between the two magnets.  The thermal energy generated does nothing else in that thought experiment.  It does not create more or less (macro) kinetic energy.  My argument in the link in the OP is a disproof (hopefully) of the Law of Conservation of Energy and not an attempted disproof of the Second Law of Thermodynamics (although I suspect the latter must also have exceptions, just not in the argument I made, too).

I hope that’s clear.  If not, please let me know.

So, while I suspect your two balls in a box example is beyond the scope of what I originally posted in this thread, I’m interested.  I’d like to understand it and discuss it.  So, please, review my drawings below and see if I’ve understood or not your proposal.  And, then, we can talk about the specifics of it.

Take care!

---


ayeaye,


I understand that in your “3/4” you have a row (a cured row) of multiple magnets and in my drawing S in Reply #11 I show only one magnet.  But I think the dynamics would be just the same.

Consider, for the ease of analysis, a straight magnet and a straight row of magnets with no gaps and a straight row of magnets with gaps (drawing N below).

In all three cases, the greatest point is repulsion is on the right end and the greatest point of attraction is on the left end.  In the case of the single bar magnet there are no other points of repulsion or attraction in between.  In the case of row of magnets with no gaps, theoretically, there are no points of repulsion or attraction in between (if all the magnets are perfectly aligned, but in the real world there will always be some physical imperfections and so there will be some (very minor) repulsions and attractions in between).  And in the case of the row of magnets with gaps there will definitely be points of repulsion and attraction in between, but these will be minor relative to the repulsion at the right end of the row and the attraction at the left end of the row.

(I make the claims that I do in the above paragraph based on years and years of playing around with magnetic configurations like this in an attempt to find a dynamic that would then lead to a successful perpetual motion machine.  I never did.)

If I understand you correctly, it is your proposal that with the gaps in in “3/4” each individual magnet in the row gives a separate push on the disk as they pass by the external fixed magnet.  It is my contention that the gaps, while they do have some impact on the system, do not push the disk further along.  It is my contention that whether the gaps are present or whether you did the same thing with one magnets or with a row of magnets with no gaps, you would get the same results as you do with the gaped row (curved gaped row) of magnets in “3/4.”  It is my contention that if you have one magnet the same (or similar) length as the row of gapped magnets that you’d get the same (or similar) distance clearance in the end in both cases (and that it is not the individual separated magnets that each give the moving body a separate and additional push, but rather the repulsion and attraction at the right and left ends of both systems).

Again, I hope you don’t mind me offering an alternative theory for your results in “3/4.”  We are on, and beyond, the borderlines of accepted science and so we should expect (just as there is within the borderlines of accepted science) lots of disagreements!  I love your design.  I never tried building your exact model.  But I did work with similar ideas.  (And perhaps you can find the exception, if there is one, in this area that I was never able to find!)

Take care.

---


- Zet



Title: Re: A possible violation of the Law of Conservation of Energy
Post by: ayeaye on April 19, 2015, 02:19:42 AM
Zetetic,

All i know is the experiment i did, and only that way there was overunity. It is difficult to make a curved continuous magnet for a disc. And the magnetic field in a curved magnet is not exactly continuous, there are like small poles in between.

It is the most important that the rotor magnets in that experiment are small and weak, and the stator magnet is strong. Because if the rotor magnets are strong, then they kind of interact too much with each other, and the results are not as good. Not sure but, it worked the best with exactly these magnets, and these were very small rectangular ceramic magnets.

As much as i know, they don't sell so small ceramic magnets anywhere. Maybe one may try with small neodymium cube magnets, they sell them even as small as one millimeter, and these should be weak enough in spite that they are neodymium. But i don't know whether that works.

But theoretically, magnetic field is not a symmetric field. Like if we draw a line between the two poles of a magnet, at one side there is the north pole, and at the other side there is the south pole. The direction of the filed lines at both sides of that line is therefore different, and the field is thus not symmetric. In every asymmetric field, there should be a path through it, so that when an object which attracts to that field moves by that path, the field does work, and the object gets additional kinetic energy. This is just mathematically so, not depending on where the energy comes from. Where the energy comes from is the matter of physics, but there must be a solution when the field is asymmetric, and thus can do work. Or there should be a configuration of the field possible, so that a pole of a magnet can go through the magnetic field, and the magnet gets an additional kinetic energy by that.
Title: Re: A possible violation of the Law of Conservation of Energy
Post by: Zetetic on April 19, 2015, 05:11:40 AM
ayeaye,


Okay.

Perhaps we have finally found my confusion with your proposal.

“The direction of the filed lines at both sides of that line is therefore different, and the field is thus not symmetric.” – ayeaye

The convention is to put little arrows on the illustrations magnetic lines of flux thus indicating a “flow” from the North Pole to the South Pole.

But this can be misleading.

It is only by convention that the arrows go from North to South.  The convention could just as easily be from South to North.  And, there is no “movement” or “flow” along these lines (as the arrows would seem to indicate).

“By convention, the field direction is taken to be outward from the North Pole and in to the South Pole of the magnet.”

http://hyperphysics.phy-astr.gsu.edu/hbase/magnetic/elemag.html

“By convention, we say that the magnetic field lines leave the North end of a magnet and enter the South end of a magnet.”

https://www.kjmagnetics.com/glossary.asp

“However, magnetic flux does not actually flow from the north to the south pole or flow anywhere for that matter as magnetic flux is a static region around a magnet in which the magnetic force exists”

http://www.electronics-tutorials.ws/electromagnetism/magnetism.html




If you’re thinking that there is a “flow” from North to South, then your drawing and your design in “Field lines chain motor 4/4” now makes sense (to me).

If field lines “flowed” (as they seem to be shown to do in all of the drawings of them) then, yes, there would be “no repulsion” on the left and right ends of the magnet in your drawing as you have indicated.  And if field lines “flowed” then your circle of magnets (“Field lines chain motor 4/4”) would each pass by the fixed in place magnet, one after the other, and the disk would spin forever.  It would work.

Is this what you are/were thinking?

If so, this is where you and I disagree.  I say that the magnetic field is symmetrical (even though, by convention, the field lines always point from North to South).  And you can easily test this.

You already have the magnets and the rotating disk.  If I’m right and you set up “Field lines chain motor 3/4" exactly as you did in that video, but this time flip all of the magnets around so that the North and South poles are in the opposite positions, then you will get the same results (the same escape on the left end).  If you’re right (again, if I now understand you correctly) then you will not get a similar result with the poles of the magnets (and thus the field lines) now reversed.


Do I finally understand your proposal?  Or, am I still not getting it?

Please let me know!

Take care!




- Zet


(PS:  Any more thoughts about what I’ve said about the “Law” of Conservation of Energy?)


Title: Re: A possible violation of the Law of Conservation of Energy
Post by: ayeaye on April 19, 2015, 01:07:08 PM
Sorry, i think i caused a confusion, and couldn't explain it well. I don't know what i were thinking. So i shall try once again.

Magnetic field is asymmetric. If you look at the field lines around a single pole, then they are not evenly distributed, neither are all of them straight. This is not a symmetric field. Like you can draw a line through one pole of a magnet, so that on one side there are more field lines than on the other. At some distance from the pole at least. It is easy to find a configuration of the field where the field lines on one side of the pole are straight, and on the other side they are bent. This is also not a symmetric field.

I don't know whether asymmetric is the best word, i use it just because i find no other. Maybe it's better to say uneven field, but an uneven field where you always in some place can find asymmetry.

Every asymmetric field can do work. To illustrate this, say if gravity were uneven, that is in some places were more gravity than in other places. Then we move an object horizontally. In the area where is more gravity, it more easily goes down, and in the area where is less gravity, it more easily goes up. That way we can easily create an engine which provides energy only because of the field of gravity is uneven. But gravity is perfectly symmetric everywhere, so we cannot create such engine.

But magnetic field is asymmetric. And this means that with some configuration of the magnetic field, we can enter the field of a pole where there are less field lines, and thus less repulsion, and we can move away from the field where there are more field lines, and more repulsion.

I hope that this helped to explain what i mean by asymmetric field, why magnetic field is asymmetric, and why an asymmetric field can be made to do work. That is, how we can extract energy only from the asymmetry of the field.

I say again that this does not necessary mean violation of the conservation of energy. This may also mean that the energy comes from some unknown source.

I also say one more thing. Why an overunity device cannot be very simple. Because the nature is not always simple. For example the magnetic field, one can see that its configuration is mostly not the simplest possible. And an overunity device cannot be simpler than nature. This does not necessarily mean very complex, but it also doesn't mean the simplest one can think about.
Title: Re: A possible violation of the Law of Conservation of Energy
Post by: Zetetic on April 19, 2015, 08:51:46 PM

ayeaye,


Cool.

The confusion is mine.  I have a certain understanding of magnetism and magnetic interaction.  And, more and more it’s becoming clear to me, that you think about magnetism and magnetic interaction is a totally different way.  And the confusion is that I keep applying my understanding to your words and the two don’t mesh.

So, in my reply to your Reply #17 I’m not going to do that.  I’m going to slow down and go through your post step by step.

(If I put in the time to really vet your words, I hope you respond.  In other words, “please don’t leave me hangin’.”)




One.

“Magnetic field is asymmetric. If you look at the field lines around a single pole, then they are not evenly distributed, neither are all of them straight. This is not a symmetric field. Like you can draw a line through one pole of a magnet, so that on one side there are more field lines than on the other.” – ayeaye

There are different ways to read this statement.

You are applying word/concept of “asymmetric” (“uneven”) to two different aspects of the magnetic field.  In the second sentence in the quote above you note that the field lines entering or exiting the same Pole are different from one another.  Some bend more and loop quickly back into (or out of) the pole while others take a longer more roundabout way in getting back to (or out of) that same pole, and in some places they are bunched up more and in other places they are more sparse.

Yes, I agree.

And, secondly, it is possible to (and I have done so) shape a magnet where the field lines entering (or exiting) one pole do not look like (do not “mirror”) the shape (the overall shape) of the field lines entering (or exiting) the other pole.  In the third sentence in the quote above you are using the word/concept “asymmetrical” (“uneven”) to the fact that the field lines at the two different poles can be shaped differently.

I also agree with this.



(However, you also seem to be suggesting that there can be more field lines at one end of the pole than at the other.  Perhaps, if you are also thinking that each line can be of different strength.  I’m not sure what to make of this yet.  So ... if it’s okay, since I’m not quite sure how to responds to this ... let’s move on.)

Did I (do I now) understand your meaning of “asymmetric” (“uneven”) as you’ve used it?

Yes?  No?




(BTW:  Have you ever played around with rubberized magnets? There are the flat thin kind that refrigerator business cards come on but you can also get them in long wide sheets (2 in. wide, 5 or 6 feet long, and 1 mm. thick, approx.), but there are thicker rubberized magnets too (not quite a square 1 cm. all the way around, and 5 or 6 feet long, approx.)  I once paid a couple of hundred bucks to a company in China to make me a few wedge shaped magnets out of ceramics.  This was in the early days of my pmm quest and I hadn’t found out about rubberized magnets yet.  You can cut them up into whatever sizes you want and stack ‘em and glue ‘em and whatever in all different kinds of exotic shapes (like a wedge).  And they only cost a few bucks.  And so you don’t have to do the stupid think I did and spend a couple of hundred bucks, instead of five bucks, to get a wedge shaped magnet.  Anyway, this is a total tangent.  But, if you haven’t come across these yet, and if you’re into exotic shaped magnets (such as the kind where the shape of the field at one pole does not mirror the shape of the field at the other pole), then I thought you might be interested.)



Two.


“Every asymmetric field can do work.” –ayeaye

Yes.

Where the field lines are closest together a metal ball will roll.

If you take a regular old ceramic bar magnet from the hardware store and if you pop a metal ball bearing down one of the pole surfaces it will immediately move from the mid region to the edge.  It is an “asymmetric” (“uneven”) field of lines of flux and the metal ball bearing moves away from the lesser amount lines of flux per given area towards the greater amount of lines of flux per that same given area (from the middle to the edge).

And so “work” (in the technical Physics meaning) is done on the metal ball by the “asymmetry” (“unevenness”) in the field.

I believe we agree.  (yes? no?)


Three.


“To illustrate this, say if gravity were uneven, that is in some places were more gravity than in other places. Then we move an object horizontally. In the area where is more gravity, it more easily goes down, and in the area where is less gravity, it more easily goes up. That way we can easily create an engine which provides energy only because of the field of gravity is uneven” – ayeaye


Okay.

I think you have missed one step of logic in the above.  I don’t think it’s a gap in your reasoning abilities (if you don’t mind me analyzing your reasoning and logic), but rather I think you have just made a simple oversight.

I do it all the time.

Say I have a skateboard ramp (which is a horizontally extended U shape) in my backyard and it spans the distance of my backyard (in the north to south direction).  And say gravity is stronger at north end of my backyard and weaker at the south end of my backyard.  (My backyard is also in a frictionless world.)

I drop in on the north side of the ramp.  As travel down the ramp, gravitational attraction accelerates me.  I then cross the horizontal part of the ramp (in this frictionless world) to the south side of the ramp.  And there I start to travel upwards.  As I do so, gravitational attraction decelerates me.

And since gravity is stronger at the north end of the ramp and weaker at the south end of the ramp, the speed I gain while going down on the north end is greater than the speed I need while going up on the south end to reach the height where I started (on the other end).

However, in the above skateboard ramp scenario, I will not reach a greater height on the south side.  In the above skateboard ramp scenario, I will reach the same height as I started.

The missing piece of the analysis is that as I move horizontally from the greater gravitational field (north end) to the lesser gravitational field (south end).  I am moving against (in the less favored direction) and across an “asymmetric” (“uneven”) gravitational field and this will slow me down.

When you write (in the above quote) “Then we move an object horizontally” I get the impression that you are thinking that there will be no deceleration as the body moves horizontally.  Yes?  No?

There will be deceleration.  And, now, after we have talked back and forth for a few times over a few days, I have the sense that your logic (your “reasoning capacity”) does recognize this as in fact true ... but/and it was rather just an oversight on your part to not notice this fact before.

Yes?  No?

And the same thing is true with a magnetic field and the asymmetries found within it.

“And this means that with some configuration of the magnetic field, we can enter the field of a pole where there are less field lines, and thus less repulsion, and we can move away from the field where there are more field lines, and more repulsion.” –ayeaye

Again, I think you have the same oversight here.  If a magnet has a enough velocity to push it ways into another magnet’s repulsively aligned magnetic field a certain distance and where the repulsion is weakest, and then the moving magnet moves horizontally (so it remains at that same certain distance) along that field to where the repulsion is greatest, and then is repulsed, it will reach the same velocity as when it started (before encountering the magnet), in a frictionless world.  The, what I believe is the, overlooked step is that it loses velocity as it moves horizontally from the weaker repulsion to the greater repulsion and so when the greater repulsion pushes it away it does so with the moving magnet moving more slowly at the start, and so the greater acceleration will get it back to (only get back to) the same velocity it was at at the start (and not something greater).

Yes?  No?



Four.

“I say again that this does not necessary mean violation of the conservation of energy.” – ayeaye

If the skateboarder reached a higher height at the end than at the start or if the repulsed magnet ended up with more velocity at the end than at the start (more “gravitational potential energy” and more “kinetic energy”, respectively) then these would both, yes, definitely be violations of the Law of Conservation of Energy.

The Law of Conservation of Energy tells us that energy can change forms but the total amount of energy remains the same (in a closed system).

So, if we make our system the whole Universe (everything) then we have a closed system (there is nothing beyond everything to interact with our system and so it is a “closed system”).

And in this closed system (the whole Universe) there is a skateboarder on the skateboard ramp in my back yard.  He drops in at 10 feet.  The gravity at the north end of the ramp, where he dropped in, is greater than the gravity at the south end.  He rolls across the horizontal part of the ramp and then up the south side curve and “vert.”  He rolls up the ramp and he comes to a stop.  (This is a frictionless world.)

If he comes to a stop at a height higher than 10 feet (the height from whence he started) then there is more “gravitational potential energy” in the end than at the start.

And, so, the total amount of energy (including this now greater amount of gpe) within our closed system (the Universe) has increased.  And the Law of Conservation of Energy tells us that this cannot (never ever, not even one time) occur.  And so, if this were to occur, it would be a violation of the Law and it would, thus, disprove this Law.

Agree?  No?



Five.

Yeah.

A successful pmm might be simple or it might not be simple.

We are in total agreement here.




So ... please reply.  Please let me know what you think.

It has been fun discussing pmm and the Law of Conservation of Energy with you.  I hope it continues.

Take care!


- Zet




(PS:  allcanadian, ... where you at?)


Title: Re: A possible violation of the Law of Conservation of Energy
Post by: ayeaye on April 20, 2015, 02:23:30 AM
Zetetic,

I only talked about a single pole. At the other pole, like in my experiment, the situation is reverse, more attraction when entering and less when leaving. So when the magnet gets more kinetic energy after going over the first pole, then it also gets an additional kinetic energy before going over the other pole. So there we may look at only one pole, what happens at the other pole is reverse.

I meant asymmetry in the sense that, if we draw a sphere around the center of the field, and we draw a plane through the center of the field, then at one side of that plane more field lines cross that sphere, than at the other side. A sphere so that the radius of the sphere is the distance at which the object, such as a pole of another magnet, passes the center of the field (center of the pole). Then when the field lines bend, at that distance there are more field lines at one side of that plane than at the other side of that plane. This description may be clumsy, but this is by now the only way to say what i mean.

Rubberized magnets is a good idea. They may be a great substitute of the small rectangular ceramic magnets in my experiment. Because these magnets must be weak, but almost all magnets which they sell are stronger.

A metal ball has a potential energy, and when it falls to the magnet, its potential energy becomes zero, so metal ball falling to the magnet is not overunity.

In the gravity example, it was not a missing part of my analyze, but rather that i thought about a different way of using the energy than you. I thought about a kind of machine, using a vertical piston on spring or such, so that repeatedly moving into the area of higher gravity and out of there, this machine can constantly generate energy. Your skateboard example was maybe better. But also if we have a large vertical wheel with a single weight on it, and this wheel is positioned so that half of it is in higher gravity and the other half of it is in lower gravity, then this wheel will continuously rotate. Provided that the difference of gravity is great enough, even if there is friction.

I meant when there are less field lines at the distance from the center of the field at which the pole of the other magnet enters the field. When the field lines are bent, then it may also be because of that.

> The, what I believe is the, overlooked step is that it loses velocity as it moves horizontally from the weaker repulsion to the greater repulsion and so when the greater repulsion pushes it away it does so with the moving magnet moving more slowly at the start, and so the greater acceleration will get it back to (only get back to) the same velocity it was at at the start (and not something greater).

No. It certainly loses some speed when it goes into the repulsion at one side of the pole, but its speed increases more than that loss of speed, at the other side of the pole, when the field is asymmetric. Provided that there is no friction or the asymmetry is great enough.

The mere fact that there is overunity, which means that the output energy is greater than the known input energy, is not enough to say that there is a violation of conservation of energy. Because some energy may come from an unknown source.
Title: Re: A possible violation of the Law of Conservation of Energy
Post by: Zetetic on April 20, 2015, 04:55:59 PM

hey ayeaye ...


“I meant asymmetry in the sense that, if we draw a sphere around the center of the field, and we draw a plane through the center of the field, then at one side of that plane more field lines cross that sphere, than at the other side. A sphere so that the radius of the sphere is the distance at which the object, such as a pole of another magnet, passes the center of the field (center of the pole). Then when the field lines bend, at that distance there are more field lines at one side of that plane than at the other side of that plane. This description may be clumsy, but this is by now the only way to say what i mean.” –ayeaye



Is the moving object moving around the curved sphere?

That’s all I got.

I read the above quote three times.  I don’t think I understand much of it.  And, therefore, only one small question is all I could come up with.

The drawing in one of your videos, I believe it was in 4/4, really helped me understand your machine.

Could you please draw out the concept in the above quote?  That would be cool.

“I only talked about a single pole. At the other pole, like in my experiment, the situation is reverse, more attraction when entering and less when leaving. So when the magnet gets more kinetic energy after going over the first pole, then it also gets an additional kinetic energy before going over the other pole. So there we may look at only one pole, what happens at the other pole is reverse.” – ayeaye

Maybe I’ll understand this better ... after seeing the drawing.  Right now I feel like I’m applying my understanding about magnetism to your words about magnetism ... and that leads us straight to confusion.

So ... I’ll wait for the drawing (... if there is one coming ... ? ... I hope).

“Rubberized magnets is a good idea. They may be a great substitute of the small rectangular ceramic magnets in my experiment. Because these magnets must be weak, but almost all magnets which they sell are stronger.” –ayeaye

The important thing to remember when buying rubberized magnets is that most commercial applications prefer to use such magnets with multiple poles per side (“multipole magnets”).  They do sell rubberized magnets that has one pole on one side the pole on the other side (and they come in the dimensions I listed above ... or at least they did years ago; “monopole”).  And our type of type of pmm experiments typically call for “monopole” rubberized magnets cut up and reconfigured.

And, yes, the rubberized magnets I used were much weaker than a regular old ceramic magnet.

Cool.

“In the gravity example, it was not a missing part of my analyze, but rather that i thought about a different way of using the energy than you. I thought about a kind of machine, using a vertical piston on spring or such, so that repeatedly moving into the area of higher gravity and out of there, this machine can constantly generate energy.” – ayeaye

My skateboarding friend agrees to do me a favor.  (I’m an old man.  If I tried to do this, I’d break my legs.)  He goes to the store and buys a very large industrial spring.  He places in my back yard, on the north end of my backward, by the skateboard ramp.  He then climbs up to the top of the ramp, without his skateboard, and jumps off from the 10 feet and lands on the large industrial spring.

The spring compresses and he locks it is place.  The spring is then rolled across the grass in my backyard, horizontally to direction of the gravitational field, to another friend waiting at the other end, the south end, of the ramp.  She sets the compressed spring on the ground and climbs on top of it.  She and the other skater both weight the exact same amount.  She then unreleases the compressed spring and she is set into motion upwards.

Where she is, the south side of my back yard, the gravity is weaker than where the spring was compressed, on the north side of my backyard. 

And so ... she will reach a higher height.

In the end (in our closed system of the Universe) there is more gravitational potential energy (gpe) than at the start.

Yes.  This is true.  Is that what you are getting at with the idea of “springs”?  Yes?  No?  Something else?

If so ... energy is conserved.

It took energy to roll the grass skateboard from north (stronger graviatational pull on the board and compressed spring) to the south (weaker g pull) and this will be exactly (.... yes, exactly, not a little more not a little less ... according to the Law of Conservation of Energy ...) the same as the amount of energy in increased gpe (height of chick on the south side at the end minus height of the dude on the north end at the start).

And pushing the board and spring into motion like this means using up energy and increased gpe is an energy gain.  And ... so ... since the two amounts of energy are exactly the same (according to this well established and basic Law of Physics) and since one is a negative and the other is a positive ... then they cancel each other out in terms of the total amount of energy within the Universe.  The total amount remains the same , while some of the forms have changed.

Or ... did I totally miss what you are trying to get at with springs?

“Your skateboard example was maybe better. But also if we have a large vertical wheel with a single weight on it, and this wheel is positioned so that half of it is in higher gravity and the other half of it is in lower gravity, then this wheel will continuously rotate.” – ayeaye

I believed this.  And ... perhaps its important to believe this ... somewhere along the way.  I think I believed it for a few days ... maybe a week.  It was back in the early days of pmm for me.  After you learn the concept you can apply it in all sorts of different places.

The concept in the original skateboarding example is the same concept in the above quote as to why this does not lead to an energy gain is the same.

However, if it was a frictionless world, yes, it will spin continuously.  But there is no energy gain.  It just spins like anything would spin, in a frictionless world, forever (no gravity needed).

However, if you’re thinking about getting an energy gain from this.  Nope.  It doesn’t happen.  Again, I believed in this for a week or so.

As the weight swings along the bottom of the wheel, down towards gravity and also moving horizontally, it is both moving into the greater gravitational field of the Earth below but also into the lesser gravitational field along the horizontal, and so there will both be a tendency to accelerate and to decelerate (and one might be greater than the other but this will be balanced out by the equal imbalance in the inverse on the other, swinging up along the bottom of the wheel, side.  (And a similar inverse dynamic occurs on the top between the two sides.))

And when I realized this, lo all those many years ago, I stop believing in what I had believed in , for a week or so.

It’s a cool idea.  I love it.

“I meant when there are less field lines at the distance from the center of the field at which the pole of the other magnet enters the field. When the field lines are bent, then it may also be because of that.” – ayeaye

Yeah, it’s beyond me.  I thought I was a smart guy ... but I can’t get this stuff.

I think maybe the drawing I requested (... if you’ve got the time ...) could really help.

“No. It certainly loses some speed when it goes into the repulsion at one side of the pole, but its speed increases more than that loss of speed, at the other side of the pole, when the field is asymmetric.” – ayeaye

Right.  I did it again.

I used my understanding of the magnetic concepts we are working with with your words.  Damn it.  I did it again.

You are right to say “No” to me.

My quote (the quote of me) in your last response was me thinking of a symmetrical magnetic field.  So ... I stand by what I wrote.  What I said is true when dealing with a symmetrical magnetic field.  (You can have asymmetrical magnetic fields in terms of non mirrored shaped of fields at the poles, but I was thinking about a symmetrical field when I wrote what you quoted.  My bad.)

And, further, I think we need to talk more about your design 4/4 particularly in regards to what I think I’m reading in the above quote (and also based on, comparison, with what you built in 4/4).

Are you thinking that in 4/4 you can move from one magnet to the next and not get the same repulsion resistance you got at the start (to get the magnet into its starting position) in 3/4?  If so, that’s true.

But are you also thinking that you retain in 4.4 the same counterclockwise push from the magnets (the now circle of magnets) that you got from the magnets (the curved row of magnets) in 3.4?  If so, then I don’t agree with you.

And that’s cool.  We can agree to disagree.  My opinion (... my analysis ...) of why I think you get the results you get in 3.4 are already posted in a previous post and don’t need to be repeated here.

I think when you go from 3.4 to 4.4 you lose both the resistance you felt with you hand in 3.4 and the leftward rotation you saw in 3.4 (... I should say , they both diminish not that you lose both).

But, that’s just my opinion.

“The mere fact that there is overunity, which means that the output energy is greater than the known input energy, is not enough to say that there is a violation of conservation of energy” – ayeaye

Yes it is.

“Because some energy may come from an unknown source.” – ayeaye

If you’re hinting at something like God or interdimensional beings or any other para-physics concept ... I’m down for that.

But ... the Law of Conservation of Energy is about our physical universe.  If God where to poke into our physical dimension give the skateboarder a little extra push ... so that on the other end he will go a little higher ... and thus end up with more gpe in the end than at the start, ... if this were to happen then we would no longer have a “closed system.”  Energy was introduced in from the outside.  And, so, yeah, sure if you introduce more energy into a system from an “unknown source” then there will be more energy in that system in the end.  This is axiomatic.

But maybe this isn’t what your saying?  I don’t know.  I too a shot at it.  Maybe I’ve totally misunderstood you.

I’m trying!

Take care my on line brother in the quest for pmm (or whatever term you prefer to use) ,



- Zet

 

(PS:  Sorry ayeaye, I didn’t have time to proof read it.  And I’m a terrible speller and such.)


Title: Re: A possible violation of the Law of Conservation of Energy
Post by: ayeaye on April 20, 2015, 08:47:21 PM
Zetetic,

Ok, drawings, first is about asymmetric field in general, and second is about an asymmetric gravitational field. What concerns the gravitational field, you may consider that there the sphere i talked about, is very big, so we may consider that there the field lines are parallel.

Again, this constantly rotating wheel in the gravitational field is not possible in reality, because the gravitational field is not asymmetric anywhere. This is only an abstract thought experiment to show that it is possible to extract energy from an asymmetric field, just because the field is asymmetric. Again, this is only a kind of mathematically so, in the reality the energy in that case may come from some other source, and there is no violation of the conservation of energy.

This is not about pmm (permanent magnet motors) only, this is more theoretical. When overunity is possible because the magnetic field is asymmetric, then it may be possible also elsewhere where magnetic field appears. Electric field may be asymmetric too, as magnetic field.

You should consider that my drawing ability is not good. I have a logical thinking and not visual thinking, so my visual capabilities are poor. So take them as they are.
Title: Re: A possible violation of the Law of Conservation of Energy
Post by: Zetetic on April 21, 2015, 03:45:51 AM


ayeaye,


Thank you for the drawings!


(BTW:  I like your drawings.  I always find them very helpful.)



First, if I may, let’s start with your second drawing (in Reply #21) and the concept therein.

In drawing E below, the skateboarder is pulled downward at E.1 due to gravitational attraction.  As he does so he accelerates.  And, in that same drawing, the skateboarder is also pulled downward at E.3 due to gravitational attraction.  But here, as he is moving upwards, he decelerates.

Since the force of gravity is greater at E.1 than it is at E.3, the speed that he reaches at the bottom of E.1 is greater than the speed he needs at E.3 to reach (and exceed) the height from which he started (at E.1).

I believe you and I agree about this.  Yes?  No?

However, as he moves across the horizontal (from the E.1 side to the E.3 side) he is moving against (away from) the greater gravitational field.  As he moves horizontally he decelerates.  E.2 is the exact same thing as if he was moving “up” and away from the strength of Earth’s gravitational field.  And when he moves “up” he decelerates.  Here to, E.2, he will decelerate as he moves horizontally.

So, when he starts to rise at the far end of the ramp (E.3) he starts out at a slower speed than he reached when at the bottom of first side of the ramp (E.1).

Yes, there is less deceleration when he moves up against a lesser gravitational field than there was acceleration when he moved down in a greater gravitational field.  But, he has already slowed down while moving from E.1 to E.3.

This is what I think is (this is what I have called) “your oversight.”

Yes?  No?

I get the impression that you are thinking that if you move from a greater gravitational field to a lesser gravitation field horizontally that there is no deceleration (as there is when you move from a greater gravitational field to a lesser gravitational field upwards).

Yes?  No?

And the same exact thing is true with a round wheel.

In drawing F the same thing happens.  Yes, there is more acceleration at F.1 than there is deceleration at F.3.  Yes.  But there is also deceleration as the weight moves horizontally (F.2). 

The difference between the two examples is that in the skateboard example the vertical movements and the horizontal movement are separated out.  But the same thing is happening with the round wheel.  It’s just that with the round wheel the vertical movements and the horizontal movement always occur together.

But it’s the same thing.

(BTW:  My analysis above and my analysis of the same in Reply #20 get to the basic point, but ... after thinking about it for a while, the Conservation of Energy analysis is much more complex.  If you are lower in a stronger gravitational field and higher in a weaker gravitation field you have the same amount of “gravitational potential energy.”  I’m sorry about this mistake.  I make them all the time.  But this limited (and somewhat incorrect) analysis is better for what we are talking about right now, I think.  That further complexity is not needed, and, in fact, I think right now would be a distraction.)

The wheel in your second drawing in Reply #21 will not gain in energy.  It is not the basis of an OU device.  I love it.  I’ve thought about it too.  But the mechanics needed are just not there.  There is a decrease in velocity in both the skateboard ramp and in the round wheel with weight as they move horizontally against the force (the horizontal force) of gravity.

Yes?  No?

-

I have a better understanding of your idea in the first drawing.  Thank you for that.  But, before we move onto that (which I think perhaps is the crux of your thinking behind 3/4 and 4/4) I want to make sure we first understand one another about the second drawing and what I’ve said.  (And, if so, then we can get to the more interesting stuff and your pmm attempt!)

-

Please let me know that you think.  Please let me know if I have understood you (and your “oversight”) or if I’m wrong (if I’m the one with the “oversight”) about what you are saying!

Take care.


- Zet


Title: Re: A possible violation of the Law of Conservation of Energy
Post by: ayeaye on April 21, 2015, 04:32:08 AM
Zetetic,

I don't understand why do you keep saying that the horizontal deceleration removes all the additional speed, i don't understand that.

Whether the wheel starts to rotate and how fast it rotates, only depends on the difference of the strength of the gravitational field in the areas of higher gravity and lower gravity. When that difference is great, the wheel certainly starts to rotate. Also when there is friction, so yes it can provide energy.

If this helps you to understand better, think about a vertical wheel, you put a small ball on it, and you blow air to it from above. If the flow of air is much stronger at the left side, where the gravitational field is stronger on the drawing, than at the right side, then the wheel starts to rotate counterclockwise, right? This is the same as the wheel in the asymmetric gravitational field, only the ball there is instead of the weight. This is an experiment which one can do, and see that it works. But if this doesn't help you to understand better, then better forget it, i didn't write it to add confusion.
Title: Re: A possible violation of the Law of Conservation of Energy
Post by: Zetetic on April 21, 2015, 06:16:18 AM

ayeaye,



I think I talk too much.

Okay.  My many many words don’t seem to have expressed the idea I was trying to express.

Okay.




Here is a real world experiment.  And you already have the rotating disk and the magnets.  (See the drawing below.)

On the rotating disk attach a magnet.  And then either tilt a bar magnet relative to the disk as shown in the drawing below or stack and glue a larger bar magnet and a small bar magnet together and place them relative to the disk as shown in the drawing below.

(The important thing is that the poles of the magnet on the disk and the poles of the bar magnet(s) are oppositely and therefore attractively aligned.)

There is more magnetic pull on the left side of the disk toward the bar magnets and less magnetic pull on the right side of the disk towards the bar magnets in both cases.

Yes?  No?

And this the magnetic equivalent of what we’ve been talking about with more downward gravitational pull on one side of a wheel and less downward gravitational pull on the other side of that wheel.

Yes?  No?

If I understand you correctly, you are saying that the magnet on the left side disk will spin towards the bar magnet and then back away from the bar magnet on the right side.  And it will spin farther away on the right side than it started on the left side (minus, of course, friction).

Yes?  No?



Take care and let me know,


- Zet

Title: Re: A possible violation of the Law of Conservation of Energy
Post by: ayeaye on April 21, 2015, 06:34:25 AM
The magnet on the disc moves towards the bar magnet and then stays there. No, this is not what we have been talking about. The fields there may be asymmetric, but this asymmetry is not used for overunity. What happens there is completely symmetric.
Title: Re: A possible violation of the Law of Conservation of Energy
Post by: Zetetic on April 21, 2015, 06:54:40 AM
ayeaye,


Well, I guess I really do need to slow down and ask more questions.

In the drawing below a body is able to move horizontally.  In the one case there is more magnetic pull on the left side than on the right.  The magnetic body will move to the left.  And I the other case there is more gravitational pull (more mass) on the left side than on the right.  The body will move to the left.

Yes?  No?

What am I missing?


Thanks,


- Zet

Title: Re: A possible violation of the Law of Conservation of Energy
Post by: ayeaye on April 21, 2015, 07:13:26 AM
This is not an analogy of the gravitational field, because the magnetic field and gravitational field are different. I showed the gravitational field example just as one example of an asymmetric field, and showed that asymmetric field can provide overunity because of asymmetry. But the interaction with the field happens the way that field works.
Title: Re: A possible violation of the Law of Conservation of Energy
Post by: Zetetic on April 21, 2015, 07:26:37 AM
ayeaye,


Will the skateboarder in the lower half of the drawing in Reply #26 move to the left?  Will he horizontally move towards the greater gravitational field?

If yes, then ...

Why would the same dynamic not also slow down the weight on the wheel in the second drawing in Reply #21 as it moves away from the greater gravitational field and into the lesser gravitational field?


Or, am I having ... yet another ... brain cloud?


Thanks and cheers!


- Zet


Title: Re: A possible violation of the Law of Conservation of Energy
Post by: ayeaye on April 21, 2015, 07:38:29 AM
Horizontal movement in the gravitational field slows down only because of friction.
Title: Re: A possible violation of the Law of Conservation of Energy
Post by: Zetetic on April 21, 2015, 07:43:32 AM



ayeaye,


Will the skateboarder in the lower half of the drawing in Reply #26 move to the left? 



Thanks,


- Zet


Title: Re: A possible violation of the Law of Conservation of Energy
Post by: ayeaye on April 21, 2015, 08:03:03 AM
I don't know. If it has some speed then it moves in the direction of that speed. If it has no speed then it stands still.
Title: Re: A possible violation of the Law of Conservation of Energy
Post by: Zetetic on April 21, 2015, 08:36:12 AM


ayeaye,


“If it has no speed then it stands still.” – ayeaye

Okay.

Cool.

We have found our point of disagreement.  Cool.

I say that gravity is “omnidirectional.”  And so, I say that the greater gravitational field on the left will pull the skateboarder (who starts out at a stop) to the left.

You, apparently, do not believe gravity is omnidirectional.  And you seem to think that gravity only pulls vertically (perhaps vertically to the surface of the body, I’m not sure exactly what you’re thinking).

Okay.  Cool.

I should have asked more questions at the start.  If we had figured out this difference between us, then all of my last several long winded post could have been avoided.  I suppose now they are all moot.  I was responding to your assertions with the idea that you and I both believed in an omnidirectional nature of gravity.

So, due to our different understandings of the nature of gravity, you think the weighted wheel design will produce extra energy while I think it will not.

We can agree to disagree.

Now, I would like to get back to your 4/4, but first ...

This is a thread about (or supposed to be about) my argument against (... hopefully my disproof of ...) the Law of Conservation of Energy (linked to in the OP).

We have talked a little about this Law here and there in the last few posts.

Any additional ideas on what I’ve said about that?

Please let me know!


- Zet



Title: Re: A possible violation of the Law of Conservation of Energy
Post by: ayeaye on April 21, 2015, 08:59:23 AM
Gravity always pulls down to the center of mass of the body. If the body is large, something like Earth, then we can consider that the field lines of the gravitational field are parallel, this i have already said. This is how "omnidirectional" gravity is, to the center of Earth from Paris or New York, the direction is quite different.

But Ok, maybe you thought how the real gravitational anomalies may occur, and i thought something fictional just to explain an asymmetric field. Which had to be evident because i drew all field lines vertical. In the reality the gravitational anomalies are caused by higher densities somewhere inside the body, there the transition from lower to higher gravity is most likely smooth, and the field lines may also be slightly tilted. Whether energy can be extracted from that, i don't know, likely not with that wheel, yet there may be ways how this is possible.

I gave that example of gravity just to explain things, again i didn't want to cause confusion. I did all i could do, i tried to explain as painstakingly as possible.
Title: Re: A possible violation of the Law of Conservation of Energy
Post by: Zetetic on April 21, 2015, 08:17:54 PM

If I’m in my backyard standing still on my stake board in the middle of horizontal part of my ramp, I am being pulled directly down by the gravity beneath me and I am also being pulled to the North due to the gravity there and I am being pulled to the South due to the gravity there.

Since, the force of gravity is, in fact, the same (nearly the same) in all part of my backyard, I will stay in place.

It’s not that I stay in place because the only gravity that is affecting me is that which is directly below me and pulling me straight down.  It’s that the gravity that is pulling me to the one side is equal (nearly equal) to the gravity pulling me to the other side.

And you and I seem to disagree.  And I’m okay with that ... I hope you are too.

And so, when we introduced the concept (the fictional concept for the sake of a thought experiment) that the gravity at the North end of my back yard is significantly stronger than the gravity at the South end of my back yard, given my understanding of gravity and its omnidirectional nature, that leads me to say that, now, in this situation I will be pulled to the North.

That’s it.

You seem to think that the greater gravity at the North will only pull more greatly downward on things directly above it at the North end.

So ... we disagree ...

I am eager to move on from this and to your idea in the first drawing in your last post (the idea that seems to lead to your 4/4 design).

But, before we do that, any additional thoughts about what I’ve said (and what we have seemed to disagree about) regarding the Law of Conservation of Energy and its relation to OU?



Cheers!

- Zet


Title: Re: A possible violation of the Law of Conservation of Energy
Post by: ayeaye on April 21, 2015, 08:49:46 PM
I gave a fictional example, just for showing what an asymmetric field is, assuming that all field lines are parallel. You keep saying that no, the field cannot be like this, and i made a mistake. Again i say that it was fictional, and gravitational field cannot be like this, i agree. But it was intentionally fictional, a field which cannot exist, but is simple, so that it would be easier to understand what asymmetric field is. I keep saying this and you keep insisting that i don't understand the gravitational field.

Your skateboard example is an entirely different example, as you are assuming a real gravitational field. So we don't disagree, but be are not talking about the same thing. It may not be possible to achieve overunity because of the asymmetry of the real gravitational anomalies, in the simple way that you propose, because the field configuration then is more complicated. It is easy to show a way how overunity cannot be achieved, and this does not prove that there cannot be overunity. So i cannot see any value of your example, and why you came up with this.
Title: Re: A possible violation of the Law of Conservation of Energy
Post by: Zetetic on April 21, 2015, 09:04:43 PM


Okay.


Shall we move on?





(!)

- Zet



Title: Re: A possible violation of the Law of Conservation of Energy
Post by: ayeaye on April 23, 2015, 10:31:04 AM
I posted this exact picture, with the gravity-like field, in two places where they discuss science, no negative reaction, i don't understand what is going on.
Title: Re: A possible violation of the Law of Conservation of Energy
Post by: Zetetic on April 24, 2015, 03:11:47 AM
We simply disagree.

You have proposed a hypothetical (fictional) thought experiment.

And we disagree about how that hypothetical (fictional) thought experiment would work.

The hypothetical (fictional) element is that there is stronger gravitational field on the left on a weaker gravitational field on the right.

Okay.  I’m fine with this.

However, you also seem to suggest that this gravitational field is unidirectional.  That is, it will pull on things “above” it but it will not pull on things to the “side” of it.

And this is where we disagree.

I do not agree that a gravitational field can be unidirectional.

However, if you want to add a second hypothetical (fictional) element to the thought experiment where “gravity is unidirectional” then, sure, such a device would work.

The magnetic example would also work the same as your gravity example if we also stipulate there that “magnetic attraction is unidirectional.”

So ... okay ... sure.  If gravity is unidirectional, then, yes, it would work.

Okay?

Can we move on?  Any ideas about my Conservation of Energy analysis in the link in the OP?

Take care!

- Zet

Title: Re: A possible violation of the Law of Conservation of Energy
Post by: ayeaye on April 24, 2015, 11:25:07 AM
> However, if you want to add a second hypothetical (fictional) element to the thought experiment where “gravity is unidirectional” then, sure, such a device would work.

Yes, this is what i did, and i said it too, why did it take so long for you to understand? So i think we now finally agree in this. It is possible that an asymmetric field can do continuous work, just because it is asymmetric.

> The magnetic example would also work the same as your gravity example if we also stipulate there that “magnetic attraction is unidirectional.”

No, this doesn't follow. I didn't assume that magnetic field is unidirectional in the magnetic example. I didn't assume anything there, other than that the configuration of the magnetic field, that is the configuration of the magnets, is such that the magnetic field is asymmetric enough. The way the asymmetry of the field is defined, which in magnets means asymmetry at one pole, or the same asymmetry at both poles.

We started the discussion about my experiment here, because of the theory of it. But this yes splits this thread into talking two theories of overunity, overunity due to asymmetry of a field, and overunity due to switching off of a field, like by the curie temperature. In essence both of these approaches are similar, in that both assume a certain irregularity of the field, that is either asymmetry or then a way to change the field such as shielding or switching it off. The difference is though that, creating an asymmetric magnetic field, the way the asymmetry is defined, looks like possible, but switching it off or shielding it may not be possible by any working and useful means.

What concerns the theory, it is likely not possible to make one equation and say that this equation shows that there is overunity. Like because it depends on a certain configuration of magnets to create a certain asymmetric field. In essence so far it is just saying that the overunity may be possible, and saying the theoretical reasons why, but only the experiments can finally show that there is overunity. So it cannot be shown theoretically by equation, it can only be shown by a model. But then it would still be a question how much that model corresponds to the reality.

I appreciate you talking about my experiments and the theory behind them. But please no more formal fallacies such that you are talking about a different thing than i do, and then ask whether we agree or disagree. And also imply that i am wrong. I don't know whether it's intentional or not. It is not possible to agree or disagree when not talking about the same thing. It is like i am talking about the fence, and you are talking about the hole in the fence. Then when i say that the fence is white, you disagree because the hole in the fence is not white. This is unfair and inevitably gives an impression that the intention is to offend and not discuss.

If you want to continue the discussion of your theory in this thread, then i would suggest to continue the discussion of my experiment in the new thread which i created for it, in the "mechanic" subforum here http://www.overunity.com/15729/overunity-due-to-asymmetry-of-the-magnetic-field/#.VTn_sTr52rM (http://www.overunity.com/15729/overunity-due-to-asymmetry-of-the-magnetic-field/#.VTn_sTr52rM) .
Title: Re: A possible violation of the Law of Conservation of Energy
Post by: norman6538 on April 24, 2015, 02:18:40 PM
Here are two other magnet anomalies that you might be interested in.

my pendulum that goes past its dropped point.
http://www.overunity.com/14876/permanent-magnet-ou-machine-working-today/msg414896/#msg414896
and overunity annimation.

http://www.overunity.com/14876/permanent-magnet-ou-machine-working-today/msg445683/#msg445683

Enjoy,
Norman
Title: Re: A possible violation of the Law of Conservation of Energy
Post by: ayeaye on April 24, 2015, 02:54:23 PM
Norman6538, interesting but, not in the video, its description or in your threads, is a complete technical data about that experiment. What magnets, how strong, how positioned, These experiments have to be repeatable, otherwise they are worthless. Looks like similar to mine, and the result is the same, overunity but not enough for a continuous rotation. But what is it based on, asymmetric field, changing field, shielded field? By the information provided, this is not clear, and so there is nothing more to talk about. So unless a complete information is provided, this experiment is worthless.

You may try to explain it by asymmetric field or other theoretical way by yourself, if you can. But if not, repeat my experiment, where complete information is provided, and which is also likely much more easily repeatable :) https://archive.org/details/Flcm3 (https://archive.org/details/Flcm3)

And although it looks similar, it is not clear how much energy is necessary to move the pendulum to the starting position. In spite that it looks obvious, the input and output energy cannot be compared, and complete measurement is the only thing that gives to these experiments any value. That something moves more somewhere than somewhere else, does not necessarily mean that the energy was greater, the force may be great. So the distances are not necessarily measures of the energies, unless the experiment is done so that they are.

In my experiment, the energy necessary to move the disc to the peak of the magnet, is exactly equal to the energy released when moving from that peak by the repulsion of that magnet. So the distance by which the disc moves by the repulsion of that magnet (the first magnet in the chain) is a measure of the energy necessary to move the disc to the peak, that is the input energy.

Now when the disc moves more by the chain of the magnets, starting from the same peak (and it did), then the output energy is greater than the input energy. So my experiment is two stage, first is to measure the input energy, and second is to measure the output energy. This is good because it enables to immediately measure the distances, and thus the energies. Because the energies correspond to distances.

One more thing, overunity in a magnetic field may also explain the overunity in solid state devices, but also overunity in some mechanical devices where elasticity or impact is involved, because elasticity is electromagnetic. The interaction between the molecules in a liquid is also electromagnetic. That is everywhere where magnetic field is involved. In a way they all involve a mutual interaction of moving charged particles and a magnetic field.

But whether the conservation of energy is violated, my gut feeling says to me that when there is an energy in a vacuum, which may be a chaotic movement of some particles, maybe the higgs bosons, then when some more orderly movement emerges, such as a movement in a field, then this decreases the chaotic movement in the vacuum, and thus correspondingly decreases the energy in the vacuum. Vice versa, a loss of order increases the energy in the vacuum. But it also may be that energy is transferred from one magnetic field to the other. Or it may come from something else overlooked. So i don't speculate where the energy comes from. I just said that to show that overunity does not necessarily mean a violation of the conservation of energy.
Title: Re: A possible violation of the Law of Conservation of Energy
Post by: norman6538 on April 24, 2015, 09:42:37 PM
ayeaye,   Are you referring to the pendulum? or the annimation? They are entirely different. The pendulum is only intended to wake us up out of the box/tunnel and realize that there is something happening there and the annimation will evolve into a demonstration of real overunity where more work out can be seen than work in.

And no one has ever to my knowledge even tried or asked to replicate the pendulum.
It should be a great scientific curiosity but currently curiosity is in short supply.

I have a permanent magnet addiction that I get sucked into from time to time.
Then I take a break till something starts cooking again.

Norman
Title: Re: A possible violation of the Law of Conservation of Energy
Post by: ayeaye on April 24, 2015, 10:01:28 PM
The only value of these experiments is that they can be replicated, they cannot be proved. The following is a thumbnail from my experiment, the field lines chain experiment. All the data to replicate the experiment is provided. A computer processor fan is used for bearings, it is also stuck to the table with a mounting tape. Which is not a problem, the mounting tape can be easily taken off the table, with no traces left. Everything is fixed with a mounting tape.
Title: Re: A possible violation of the Law of Conservation of Energy
Post by: Zetetic on April 26, 2015, 12:08:42 AM
ayeaye ,


“... but switching it off or shielding it may not be possible by any working and useful means.” - ayeaye


Please see the drawing below.


When a bar magnet loses its magnetic alignment, it becomes demagnetized.

If there is another magnet in the demagnetized magnet’s magnetic field, then impact of this loss of internal magnetic alignment of the one magnet (this demagnetization) takes time to cross the field to the other magnet.

And so, while the demagnetized magnet instantly loses its attraction (or repulsion if alternatively aligned) with the other magnet, it takes some time for this loss of attraction (or repulsion) to make its way across the distance to the other magnet.

This is simply how Physics works.

There is no “shielding” or other “means” included in this thought experiment.

And if one magnet is moving (and accelerating) while the other one is not, then if it is the moving magnet that is demagnetized, the acceleration immediately stops and so there is less kinetic energy in the end, while if it is the fixed in place magnet that is demagnetized, the acceleration continues for a time longer, and so there is more kinetic energy in the end.

And since both systems started out with identical amounts of energy, the Law of Conservation of Energy tells us that there must be identical amounts of energy in both systems at the end.

But this does not happen in the set up (thought experiment) I propose in the link in the OP.  (Unless, you can see something I don’t.)

And therefore the Law of Conservation of Energy has been shown to be logically flawed, it is internally inconsistent, and therefore it is false.

That’s it!


-


“... only the experiments can finally show ...” – ayeaye

I completely disagree.

Logic is logic.

If a theory is internally logically inconsistent then it is by definition false.

Experiments can always be argued about as to whether the test was done right and as to whether the results were accurately measured and so on.

But, logic is logic.

And if the Law of Conservation of Energy is internally logically inconsistent (as I believe I have shown in the argument in the link in the OP) then we know this “Law” is false to a logical certainty.  (No experiments needed.)


-


“It is like i am talking about the fence, and you are talking about the hole in the fence. Then when i say that the fence is white, you disagree because the hole in the fence is not white.” - ayeaye

Well ... I’m always open to not only the possibility that I may have gotten something wrong or that I’ve been less than clear, but I’m also open to the real fact that I will get things wrong and I will be less than clear.

I am always trying!

However, you may also want to take some time to consider if it is within the realm of possibilities that you have gotten some things wrong or have been less than clear.  Is it possible?  ... I don’t know.

You use the terms “asymmetric field” and “uneven field.”  And I’m fine with letting you define these terms as you see fit and then working with that definition.

In your Reply #17 your write “Magnetic field is asymmetric.”  And then, in that reply, you go on to show that the “magnetic field is asymmetric” by pointing out that the magnetic field lines at a single pole are unevenly distributed and that the magnetic fields lines between the two poles can differ from one another.

Okay fine.

And then in your second drawing in Reply #21, you show a weighted round wheel in a hypothetical “asymmetrical” gravitational field.  In that drawing you show the gravitational field lines bunched up on the left side of the wheel and the gravitational field lines spread further apart on the right side of the wheel (... just as in Reply #17 you talked about a “asymmetric magnetic field” with the magnetic field lines unevenly distributed).

At no point, that I can see, in text of Reply #21 (or in the text of your posts that proceeded this drawing or that immediately followed this drawing) do you clarify that you are using the term “asymmetric” in regards to a magnetic field (Reply #17) and in regards to a gravitational field (Reply #21) in two different ways. At no point, do I see you write that what you mean by “asymmetric” when it comes to magnetism is to include the idea that “magnetic attraction is omnidirectional” while, at the same time, you mean by “asymmetric” when it comes to gravity is to, differently, include the idea that “gravity is unidirectional.”

If I’m reading you posts correctly, you use the term “asymmetrical” to apply to both magnetic fields and to gravitational fields, but in the latter you mean something different than in the former (unidirectional versus omnidirectional), but you never (if I’ve read your posts correctly) actually make this distinction.

In Reply #27 you write “This is not an analogy of the gravitational field, because the magnetic field and gravitational field are different.”

Gravitational attraction and magnetic attraction are not unidirectional.

Your proposal of a unidirectional gravitational field (as part of an “asymmetrical,” unevenly dispersed gravitational field) appears to me to be nothing more than a matter of stipulation.  And I’m fine with that (for the purposes of a thought experiment).  But, I cannot then see why you or I or someone else cannot propose that same thought experiment, but with magnets, and also stipulate a unidirectional magnetic field.

You seem to be saying that there is some real fundamental difference between gravitational attraction and magnetic attraction, while then separately also hypothetically (fictionally) stipulating a unidirectional gravitational field (that you don’t seem to also think can be hypothetically (fictionally) equally stipulated with a magnetic field).

I don’t get it.

If you are being clear about all of this ... if I am just simply dumb and daft ... then I apologize.

I’ve worked hard to try to understand you.  I think your design (4/4) is cool.  I don’t think it will ultimately work (as you found out in your experiment).  But I’m willing to take the time and effort to get to (clarify) and work through your theory.

But, maybe I’m just so mentally clouded that I can’t see the clarity that you have presented.  ... I don’t know.

Again, at no time do I see you distinguish an “asymmetrical” magnetic field (your Post #17) from an “asymmetrical” gravitational field (your Post #21), in that the former obviously is not unidirectional and the latter is obviously unidirectional until I finally drilled down on the discussion and brought it to a fine point in my Reply #30 (and thereabouts).



Take care!

- Zet



(PS:  Any additional ideas on my argument against the Law of Conservation of Energy linked to the OP?)





---



norman6538 ,


Thanks for the link.  Cool model.

If I understand it correctly, you start on one side (“two o’clock”) where there are no magnets, and then you swing down to where the magnets start (“six o’clock”), and then you swing up the other side where there are more magnets until you reach the end of these magnets (“twelve o’clock”), and then you swing back down past these other magnets back to the point of greatest attraction (“six o’clock).  (The magnets between six o’clock and twelve o’clock seem to be attracted to the moving magnet but also each one individually seems to be less attracted to the moving magnet than the one magnet at six o’clock.)

Is this what is happening?

If so, this is exactly what I would expect to happen.

If you just had the moveable magnet and the magnet at six o’clock, then what I would expect that in a frictionless world the moveable magnet would swing from two o’clock (where it starts) to ten o’clock on the other side (the same distance from the fixed magnet)and then back again, and it would do this forever.  And if this two magnet arrangement were to be done in a real friction filled world, then I would expect the moveable magnet to swing back and forth for a bit and then come to a stop next to the fixed in place (six o’clock) magnet.

If I understood the mechanics of your video correctly (and there are no magnets on the one side where you start, but there are magnets on the other side between six o’clock and twelve o’clock) then I would expect the moveable magnet (which starts out at two o’clock) to reach a distance further from the point of greatest attraction (six o’clock), as it does, and then eventually end up in the end at the point of greatest attraction (six o’clock), which it does.

Did I get the set up in your video wrong?

If I did, if I’m wrong about your proposal, please let me know!

Again, cool model!



BTW:  Do you have any thoughts about my argument against (my hopeful disproof of) the Law of Conservation of Energy linked to in the OP of this thread?



Take care!

- Zet


(PS:  Could you put your animation on Youtube, so that we don’t have to download any extra files?  Thx.)



Title: Re: A possible violation of the Law of Conservation of Energy
Post by: ayeaye on April 26, 2015, 10:27:59 AM
Zetetic, i think you should explain your theory some more, your last drawing really gives no idea of the horns or tail. So you say some energy is lost because the magnetic field is switched off? How can a potential energy be transformed to another form of energy i don't well understand either. It seems that when there is a law, then the equations always have to be written so that the law is valid. Which maybe cannot be done in some cases. You may have a point, but you should explain more.

If you can draw faster with a pencil, do that and capture the image with a web cam. I think when it's easier and faster, then it's better than the high quality drawings made with some software. In some science discussion some asked me, did i draw these drawings on a napkin. Absolutely wrong, i drew them on a packing paper, because this was the only paper i had under hand. Use simple means to do things, this may make all the difference between doing and not doing. And i'm more into thinking, so these practical things are rather a nuisance.
Title: Re: A possible violation of the Law of Conservation of Energy
Post by: Zetetic on April 26, 2015, 09:22:44 PM


“... i think you should explain your theory some more” - ayeaye


Gladly, I will.  Thank you for asking!

-

“... your last drawing really gives no idea of the horns or tail.” - ayeaye


Yeah, that was less than clear.  The drawing in my last post here in this thread was one of the four drawings that are embedded in my argument in my Science Forums argument against the Law of Conservation of Energy (linked to in the OP here).

Yep, I guess by just reposting it here (with no context) was not helpful.  (Also, it somehow got corrupted when I posed it, unlike all of the other drawings I have posted on this forum in this thread that showed up just fine.  I don’t know, but again, not helpful.)


-


“How can a potential energy be transformed to another form of energy i don't well understand either.” - ayeaye

If I’m standing in my backyard and I throw a ball up into the air, it slows down as it rises and eventually comes to a stop.

When the ball left my hand it was in motion.  This motion is a form of energy (“kinetic energy”).  And this motion is a certain amount of kinetic energy, say 10 units of kinetic energy.

After the ball rises and then comes to a stop, those 10 units of kinetic energy are now gone.  However, the Law of Conservation of Energy tells us that the total amount of energy within a closed system (such as the Universe, everything) will always remain constant.  But we are now missing 10 units of energy (kinetic energy) that was in our closed system but is now gone.

Energy is conserved, in this case, because as the ball rises and slows down (an increasing loss of kinetic energy) there is a corresponding and equal gain (assuming no friction) in “gravitational potential energy.”  When the ball is closer to the ground it has less potential energy and when the ball is farther from the ground it has more potential energy.  (A ball farther from the ground can do more “work” (in the technical Physics sense) than a ball closer to the ground and so it has more “potential” energy.)

And so, when the ball finally comes to a stop, there is a decrease in 10 units of kinetic energy and a corresponding and equal increase in 10 units of potential energy.  So the total amount of energy within the Universe remains the same (10 units of energy becomes 10 units of energy).

And now (to answer your question (... if I understand it correctly...)) as the ball at rest up in the air then falls back to my hand, that potential energy then becomes kinetic energy.   As the ball falls back down and speeds up, there is a loss of 10 units of potential energy and a corresponding and equal increase in 10 units of kinetic energy (as the now moving again ball smacks back into my hand).

Does this answer your question?  Did I miss it?  Do you already understand all of what I just wrote, but, rather, you were asking about the role of “potential energy” specifically in the context of my moving and demagnetized magnets thought experiment?

In my thought experiment in “Post #21” in the thread in Science Forums (liked to in the OP of this thread here) I take care to not talk about changing amounts of potential energy as the source for the violation of the Law of Conservation of Energy in my argument.  (The key word here is “amounts.”)

The concept of “how much potential energy” in mainstream Physics is a subtle and slippery one.  I can explain it here in this thread it you’d like (just ask).  But to go further into it now, would be a long discussion that ... I believe ... might be beyond the scope of your question (... I don’t know).

In my argument against the Law of Conservation of Energy there is , yes , definitely a loss of potential energy between the two magnetically attracted magnets when one of the them becomes demagnetized (“switched off”).  And this is definitely a key factor in that thought experiment.  But the amount (“how much”) potential energy lost is purposely avoided.  There is potential energy between the two magnets just before the moment of demagnetization and how much is irrelevant to the argument.

(The main point when it comes to potential energy in that argument of mine, is that the loss of mutual attraction takes time to cross the distance to the other non-demagnetized magnet.  But, I’ll explain this more below.)

I hope this has been helpful and addresses your question.  If not, please let me know!

-

Okay, back to the main point in your last post (“... i think you should explain your theory some more ...”).

The argument that I make in Science Forums is very carefully done.  I took about a week or so (not non-stop but every now and then and here and there) to write and rewrite and rewrite it.  I wanted to get the language and concepts clear and correct.

But, I can here give a more casual rendering of the same case.  And this may be helpful.

Say, I have two magnets on my desk and they are magnetically aligned.  I’m holding one in my hand and so it is fixed in place.  And say I glue the other magnet to the back of a little plastic toy car.  The magnet and toy car are also on my desk, but they are free to move.

The two magnets are in one another’s magnetic fields.  There is a tendency for them to move towards one another.  The one on the back of the toy car can and will while the other one in my hand is held in place and prevented from doing so.

As the toy car and magnet move towards the one in my hand they accelerate.  The faster they more the more “kinetic energy” there is.

At this point energy is conserved.  The increase in kinetic energy (motion) is matched by an equal and corresponding decrease in the potential energy between the two magnets due to their mutual attraction.  (5 units of energy gained comes from 5 units of energy lost).

At this point, all is good in the world (of mainstream Physics).

Before the two magnets (the one on the toy car and the one in my hand) collide, the magnet on the toy car is demagnetized (its internal magnetic alignment becomes unaligned).  How this demagnetization occurs is not so important in this more casual description.  For the sake of this description of my thought experiment, it just does.  (There are different ways to demagnetize a magnet and I’ll address that separately below.)

When the moving magnet on the back of the toy car becomes demagnetized it is no longer mutually attracted to the magnet in my hand.  (It’s no longer a magnetized magnet.)  And, so, it stops accelerating.

Assuming no friction, the now demagnetized magnet and toy car will continue to move at the same velocity that they had at the time of the demagnetization.

Now, if instead, when the two magnetized magnets (the one on the toy car and the one in my hand) are the same distance apart from one another in the scenario just described above, the magnet on the toy car remains magnetized while the magnet in my hand is demagnetized, the magnet on the toy car (and the toy car) will continue to accelerate for a moment longer.  When the magnet held still in my hand is demagnetized it is instantly no longer mutually attracted to the magnet on the toy car.  However, this loss of mutual attraction is not instantly “communicated” across the field to the other still magnetized magnet (the one on the toy car that is accelerating).  And so the magnet on the toy car will continue to accelerate for a moment longer (it will get faster), and so in the end, in this second scenario, we end up with more kinetic energy than in the first scenario (where it was the moving magnet that was demagnetized).

Okay, that was the practical set up of the thought experiment.  Now here is the theoretical importance.

The Law of Conservation of Energy states that “in a closed system, while energy can change forms the total amount of energy remains the same.”

In the first scenario (where the moving magnet is demagnetized) there is less kinetic energy in the end.  And in the second scenario (where the fixed in place magnet is demagnetized) there is more kinetic energy in the end.

So, if the total amount of energy within the Universe is to remain the same in both cases, then in the first scenario there must be more of another form of energy (to make up for the lesser amount of kinetic energy) and in the second scenario there must be less of this other form of energy (to counter the greater amount of kinetic energy).

In my argument linked to in the OP, at this point I leave it as a question:  “What is it?”

Because I believe that there is no answer to this question.  I believe that we end up with two different total amounts of energy within the closed system of the Universe in the two different cases.  I believe that there is not an offsetting increase and decrease in another form of energy between the two cases which keeps the total amount of energy within the Universe constant.

And, so, I believe this thought experiment shows a violation of the Law of Conservation of Energy, and therefore this “Law” has been demonstrated to be false (and to a logical certainty).

That it!

(I realize this “casual” description is very long.  It is hard to make all of the relevant points in a shorter form.  That’s why I took so long and worked so hard on the argument linked to in the OP.  It took effort to get it down to two pages.)

I hope this helps!

Let me know if it doesn’t (or it if does)!


-


There are different ways to demagnetize a magnet.

(allcanadian alluded to this in his Reply #13; “... generating heat ...”, “... impact in itself ...”.)

Here is a link to a Wikipedia article on demagnetization:

http://en.wikipedia.org/wiki/Magnetization#Demagnetization

You can demagnetize a ferromagnet by raising its temperature above a certain amount, and you can demagnetize a ferromagnet by striking it hard.


-


I hope this helps make my point (argument) clearer.  If not, please let me know.  I apologize for the lengthy length of it!  It’s harder to keep something short (and hit every point necessary) than it is to let it flow longer (while also hitting every point).  The argument linked to the OP is short and hits every point.

And thank you for continuing the conversation about my argument against (hopefully my disproof of) the Law of Conservation of Energy!

Let me know what you think!

(allcanadian , norman6538 , any more ideas?)


Thank you and take care!


- Zet




(PS:  I have spent a lot of time in Microsoft Paint.  And so it’s real easy for me to make my drawings there.  Whether a drawing is hand drawn or computer drawn, to me, makes no difference.  What matters, to me, is their clarity.  So, if you choose to hand draw and I choose to computer draw, I really see so difference.  Cheers!)




Title: Re: A possible violation of the Law of Conservation of Energy
Post by: ayeaye on April 26, 2015, 10:32:15 PM
Ok, so your argument is like this. A fixed magnet, and a moving magnet, attracting each other. You do two experiments, in one experiment you demagnetize the moving magnet, and in the other you demagnetize the standing magnet. The input energy is the same. Then, as you argue, in the second case the output energy is greater, because the demagnetization of the standing magnet influences the moving magnet with a delay.

What about to modify the experiments, so that in both cases we demagnetize the standing magnet, only at different times. Then too i guess, the input energy is the same, the energy necessary for demagnetization is also the same, but the output energy is different.

As i see it, when assuming that there is a conservation of energy in that experiment, then at the moment of demagnetization, the potential energy of the moving magnet has to be transformed to some other form of energy. So when the kinetic energy of the moving magnet after the demagnetization will be less, the energy in some other form, like vacuum energy, will increase by the same amount.

I understand how a potential energy in a gravity transforms into a kinetic energy. But i don't see how that same potential energy can transform into another form of energy, such as heat, electromagnetic energy, vacuum energy, etc. This is why i asked about transforming potential energy to another form of energy.

Because as much as i can see it now, the potential energy in your experiment has to be transformed into another form of energy, at the moment of demagnetization. As there seems to be no possible way how this can happen, there should be a violation of the conservation of energy. It is possible that i overlook something, but this is how i see it by now.
Title: Re: A possible violation of the Law of Conservation of Energy
Post by: Zetetic on April 27, 2015, 01:45:20 AM

“Ok, so your argument is like this. A fixed magnet, and a moving magnet, attracting each other. You do two experiments, in one experiment you demagnetize the moving magnet, and in the other you demagnetize the standing magnet. The input energy is the same. Then, as you argue, in the second case the output energy is greater, because the demagnetization of the standing magnet influences the moving magnet with a delay.” - ayeaye


Yes.  Exactly.


-


“What about to modify the experiments, so that in both cases we demagnetize the standing magnet, only at different times. Then too i guess, the input energy is the same, the energy necessary for demagnetization is also the same, but the output energy is different.” -ayeaye



Right.

We have two systems that start out identical in terms of energy (see the drawing below).

In both systems there are two magnetically aligned magnets the same distance apart from one another (same potential energy).  Both magnets in both systems are at rest (no kinetic energy).  And in both systems there is a chemical heat pack (same chemical potential energy).

The magnets on the right in both systems are held in place, while the magnets on the left in both systems are allowed to move.

The two left magnets accelerate towards the fixed magnets on the right (there is an increase in kinetic energy and an equal decrease in potential energy; energy is conserved).

When the one magnet in the one system is a certain distance away from the fixed magnet, the chemicals are exposed to one another and it becomes demagnetized.  It immediately stops accelerating.

In the other system the other moving magnet continues to move and accelerate.  And then, at a point where it is moving faster and is closer to the fixed in place magnet, this moving magnet is demagnetized.  And it then stops accelerating.

So, in the end, in the one case there is less kinetic energy and in the other case there is more kinetic energy.

(The same thing can be done with demagnetizing the fixed magnets, and the analysis is essentially the same.)

If there is not also a corresponding greater amount of another form of energy in the case with less kinetic energy and a corresponding lesser amount of this other form of energy in the case with more kinetic energy, then the logic of the Law of Conservation of Energy has been violated and it has been proven false.

Yep.

And it may be the case that this scenario is also, in fact, a violation of, and disproof of, the Law of Conservation of Energy.

However, in this scenario there is some logical wiggle room.

It seems counterintuitive at first but the demagnetized state is the “higher energy state” and the magnetized state is the “lower energy state.”

If there is a ball at rest at the top of a hill it is in the “higher energy state” and if there is a ball at rest at the bottom of the hill it is in the “lower energy state.”  That is, it takes energy to move the ball from the bottom of the hill to the top, while the ball will naturally move from the top of the hill to the bottom of the hill if it is able.

Same thing with magnets.  It takes energy to take a magnetically aligned magnet and to then internally disalign it (to “demagnetize” it).  While, if able, the particles within a ferromagnet will naturally magnetically align.  And so, the magnetized state is the “lower energy state” (like the ball being at the bottom of the hill) and the demagnetized state is the “higher energy state (like the ball being at the top of the hill).

(This is called “spontaneous magnetization”:  http://en.wikipedia.org/wiki/Spontaneous_magnetization)

This can be counter intuitive because we normally think of a bar magnet as “having energy put into it” when it becomes magnetized.  And we normally think of a bar magnet as losing its magnetism over time if it is just sitting around for a long long time.  But this is not the case.  A magnet does not “spontaneously demagnetize.”  Some other force (energy) must demagnetize it.

Okay.  So why is this relevant to the thought experiment you have proposed?

Here is the “wiggle room.”

Since the demagnetized state is the higher energy state and the magnetized state is the lower energy state, that means, in this thought experiment, when the magnetized magnets become demagnetized there must (according to the Law of Conservation of Energy) be an equal decrease in another form of energy.  And here we have the increase in energy in the form of demagnetization offset by a corresponding and equal decrease in thermal energy.

Thermal energy (heat) is “micro kinetic energy;” it is movement but on a tiny scale.  And it is this microscopic movement that works against the tendency for the particles within the magnet to align with themselves.  And so, as the movement of the thermal energy moves the magnetically aligned particles into randomized disalignment, (“demagnetization”) there is a loss of this energy (thermal energy, micro kinetic energy); just as there is a slowing down (on a macro scale), a decrease in kinetic energy, when a motorcycle reaches a ramp and is redirected upwards (and to a higher energy state).  The same thing happens on a “micro” scale as occurs on a “macro” scale.

I hope all of this makes sense.  (?)  If not, please let me know!

Okay, so back to the thought experiment.

So, it takes energy to demagnetize a magnet, and in this case this energy comes from thermal energy.  In this thought experiment, when the chemicals in the pack are first exposed to one another there is a loss of chemical potential energy and there is an equal increase in thermal energy.  However, when that increased temperature then reaches and demagnetizes the magnets there is a loss in thermal energy and an equal increase in energy in the form of demagnetization.  Energy, at all times, is conserved.

Now, in this thought experiment, there is a difference between the two cases.  In the one case the moving magnet is demagnetized while further away from the fixed in place magnet while in the other case the moving magnet is demagnetized while it closer to the fixed in place magnet.  And the two magnets (the moving magnet and the fixed magnet) in the two cases are in each other’s magnetic fields.  And so, by being in each other’s magnetic fields they reinforce each other’s magnetic (internal magnetic) alignment.  And so, given that the closer together two magnets are, the greater the magnetic strength of each magnet is on the other magnet, ... this means it should take less energy to demagnetize the one moving magnet which is further from the fixed magnet (so, a lesser decrease in thermal energy) and more energy to demagnetize the other moving magnet which is closer to the fixed magnet (so, a greater decrease in thermal energy).

And so, here is the wiggle room.

In the case with less kinetic energy in the end (the further apart scenario) there is more thermal energy (less of a decrease) and in the case with more kinetic energy in the end (the closer together scenario) there is less thermal energy (more of a decrease).

And if the difference in kinetic energies between the two systems just happens to be exactly equal to the opposite difference in thermal energies between the two systems, then energy is conserved!

Now, whether or not these two differences are precisely equal is an empirical question.  Maybe they are (and energy is conserved and this Law is not violated) or maybe they are not (and energy is not conserved and this Law is then violated).

The point is, energy could be conserved.  There is a logical possibility that it could be conserved.


That was a lot of words.  I hope it wasn’t too much.


So, this is why in my thought experiment I stipulate “... when the two magnets in the two systems are the same distance apart ...” the moving magnet in the one system is demagnetized and in the other system the fixed magnet is demagnetized.

As I said, I spent a lot of time working on the argument linked to in the OP of this thread.  Something like “when they are the same distance apart” in that argument of mine might seem like some throwaway line (or throwaway concept) but it is there (they are there) for a purpose.

By setting up the argument as I did (in the link in the OP) I have eliminated this logical way of “wiggling out” of the conclusion that there is in fact a violation of the Law of Conservation of Energy.

I believe my logic in that link is very tight.  But that’s why I’m here.  For stuff (alternative possibilities and ways of looking at the issue) like this.  Thank you.  I hope you will continue to consider my argument and any alternative issues around it!  Thank you.

Please let me know if this makes or does not make sense!



-



“As i see it, when assuming that there is a conservation of energy in that experiment, then at the moment of demagnetization, the potential energy of the moving magnet has to be transformed to some other form of energy. So when the kinetic energy of the moving magnet after the demagnetization will be less, the energy in some other form, like vacuum energy, will increase by the same amount.” - ayeaye


Yep.

“Potential energy” is a somewhat subtle and slippery concept.

When I roll a ball across the floor and it comes to a stop, 10 units of kinetic energy (when the ball leaves my hand) becomes 10 units of thermal energy (when the ball comes to a stop due to friction).

The 10 units of kinetic energy was a real thing.

And the 10 units of thermal energy is also a real thing.

They exist.

However, when dealing with “potential energy” (in mainstream Physics) things are different.

When I throw a ball up in the air and it comes to a stop, 10 units of kinetic energy (when the ball leaves my hand) becomes “an increase” in 10 units of potential energy (when the rising ball comes to a stop; and assuming no friction).

The 10 units of kinetic energy was a real thing.

But the increase in 10 units of potential energy is not a thing in the same way.

Potential energy, as they say, is the amount of potential energy in a given situation relative to an amount of potential energy designated as 0.  Huh?

When the ball thrown up in the air is just leaving my hand, it could be (it could be designated) that the amount of potential energy between the Earth and the ball at this point is -10 units of potential energy.  And so, then as the ball rises and comes to a stop the increase in potential energy is from – 10 units to 0 units.

So, yes, when you have two magnets some distance apart, it seems like you have a real thing (because there really is attraction between those two magnets and if you let them go they will really in fact move towards one another).  And, yes, so it seems like if you demagnetize one of them and thus lose this real attraction that they had (this real potential energy that they had), that if the Law of Conservation of Energy is true then there should be a corresponding and equal increase in another form of energy to offset this lose in potential energy between the two magnets.

Yep.  It seems like this should be the case.

But, I didn’t write the accepted and known Laws of Physics.
 
As I said, potential energy in mainstream Physics is a subtle and slippery concept.

So, when two magnets are some distance apart and one is demagnetized and so the potential energy between them is lost, it could be that 0 units of potential energy have become no units of potential energy.  And so, energy is conserved.

(This is not my argument but the argument of mainstream Physics.)

(BTW:  There is a way to bust this argument.  The link in the OP of this thread leads to “Post #21” in a Science Forums thread.  However, the OP in that same Science Forums thread (which I wrote), I believe, shows why this subtle and slippery use of the concept of potential energy cannot hold and breaks down.  But that post (the OP over there) is so long and complicate that I don’t think anyone really read it.  That’s why I kept playing with these concepts and got the argument down to a two page, very simple and very tight (logically wise), argument in the linked to “Post #21.)



-



I hope I have answered your questions.

I hope I have addressed your concerns.

I realize there are a lot of words here.  My argument linked to is very short, but every little detail (such as “when the two magnets in the two systems are the same distance apart”) has a lot of thought behind it.   I hope you find this interesting.  I love explaining what is behind each element in that argument.


If I’ve missed your points and didn’t address them, please let me know!



Thank you for continuing the conversation about my argument against the Law of Conservation of Energy!




Take care my online overunity friend,

- Zet




Title: Re: A possible violation of the Law of Conservation of Energy
Post by: ayeaye on April 27, 2015, 03:04:07 AM
Ok, the unclear part. You say that it takes more energy to demagnetize a magnet when another magnet is near it. And the other magnet remains fully magnetized? This doesn't make much sense. But anyway, when this is so unclear, then change the experiment so that we demagnetize *both* magnets at the same time. Only in the second experiment later than in the first experiment. This demagnetization should take the same energy in both cases. And the obscurity is gone.

Or do the magnets get more field energy when they are closer together? This doesn't make much sense, and the experiments i have done with magnets don't confirm that. If i have two magnets of a certain strength and i put them together, then i get a magnet with the sum of the strengths of these two magnets. At least this is what i have seen.

Notice that i said that in case of demagnetizing the magnets, the *potential energy* of the magnets should be transformed to another form of energy. I didn't say that without a reason. Demagnetization removes the magnetic field energy, and this energy is very small btw, may be much less than the kinetic energy or potential energy. This energy transforms to heat likely.But anyway, this is a different thing, and has nothing to do with the problem.

What will be lost when the magnets are demagnetized, is the *potential energy* of the magnets, and only that. And that this potential energy is different in two experiments, this is what causes the problem. When the energy should be conserved, then the only way is that this potential energy will be transformed into some other form of energy. But that there is no way for a potential energy to be transformed into anything else than kinetic energy, this causes a paradox. And makes the conservation of energy in that case impossible in any known or unknown way.

Zetetic, Ok, a piece of iron is approaching a magnet, and the magnet shall be demagnetized at different times. Even simpler. I don't think that anyone says that an iron near a magnet makes the magnet anyhow stronger or weaker, and that demagnetizing it takes any different energy, no matter how far the piece of iron is from the magnet. I would make a drawing of the experiment below too :)

[S N]        <--        [Iron]
[___]    <--    [Iron]
...
[___][Iron]

[S N]        <--        [Iron]
[___]<--[Iron]
...
[___][Iron]

Cool thing you thought out. If there is any problem that the things there are happening too fast, and the magnet cannot be demagnetized fast enough, something, then the magnet and the iron can be placed in a heavy oil, or such, so that everything happens more slowly.

It may be that i overlooked something, but this is how i see it by now. I think that you see now that my analytical thinking is not bad at all.
Title: Re: A possible violation of the Law of Conservation of Energy
Post by: Zetetic on April 27, 2015, 09:15:22 PM




“Ok, the unclear part. You say that it takes more energy to demagnetize a magnet when another magnet is near it. And the other magnet remains fully magnetized? This doesn't make much sense. But anyway, when this is so unclear, then change the experiment so that we demagnetize *both* magnets at the same time. Only in the second experiment later than in the first experiment. This demagnetization should take the same energy in both cases. And the obscurity is gone.” – ayeaye


I am not saying that it takes more energy to demagnetize a magnet when it is in another magnet’s magnetic field, ... but I am saying that it is possible.

Imagine a magnet and a magnet and a metal ball bearing.  The ball bearing is in the magnet’s magnetic field and so the particles within it are, because of this, magnetically aligned.

If a heat pack is broken near the ball bearing, then the increase in temperature will work to tend to disalign (randomly align) the particles in the ball bearing.  In other words, it will becomes demagnetized.

This takes energy.  And so there will be a decrease in thermal energy.

If the metal ball bearing was not in the magnet’s magnetic field then the particles within it would not be magnetically aligned.  And if a heat pack was broken near it, then there would be no tendency to magnetically align within the metal ball bearing for the randomization of thermal energy to overcome.  There would be no decrease in thermal energy.

The same thing when two magnets are in one another’s magnetic field.  Just as a metal ball bearing in a magnet’s magnetic field is externally magnetically aligned, when one magnet is in another magnet’s magnetic field its own alignment is externally reinforced.

And, so, it’s possible that when a magnet is in another magnet’s magnetic field and it is demagnetized that it takes more energy to do this.

It’s possible.  And that’s the point.

I am challenging one of the most fundamental laws of Physics.  If there is any wiggle room they will take it.

And, by the way, I tested this exact proposition some time ago.

I posted, essentially, the same thought experiment that you have proposed in Science Forums.  I guessed that this is where they’d (logically) go.  And they did.

(See: http://www.scienceforums.net/topic/87342-where-does-the-potential-energy-of-a-field-reside/?p=847467 and the response that follows)

Again, maybe energy is conserved or maybe not.  I don’t know.  It is an empirical question.  However, there is logical wiggle room.  And the whole point of my specific argument (linked to in the OP of this post) is to craft an argument that eliminates all of the wiggle room and logically corners the Law of Conservation of Energy and its flaws.

Maybe if you demagnetize both magnets at the same time energy is conserved or maybe not.  (Empirical question.)  However, when you demagnetize both magnets simultaneously they are (at that moment) in one another’s magnetic field (it takes time for the demagnetization of each to reach the other) and so, I believe, this still leaves that same logical “wiggle room.”  And if there is any wiggle room, mainstream Physicists will take it!  (Whether it is actually physically true or not.)

-

If the heat pack is closer to one magnet and further from the other magnet, then when the heat pack is broken the closer magnet will become demagnetized before the other.

The loss of mutual attraction will cross the distance between the two magnets far faster than the increased thermal energy will then dissipate across the distance between the two magnets (and then demagnetize the second magnet).  This is the time period where we get two different amounts of kinetic energies, depending on whether it is the moving or the fixed in place magnet that is demagnetized first.

(In the “casual” description of my thought experiment here, I left out that the second magnet will eventually become demagnetized because it is superfluous.  However, you noted its absence, as I’m sure others will too, and so in the actual argument (in the link in the OP in this thread) I take the time to anticipate all such things and include them (for the sake of clarity).  In that statement of the thought experiment, I note that both magnets will be demagnetized in the end.  And if you don’t believe me that the retreating magnetic field lines will cross the distance between the two magnets much much faster than the heat dissipates across that distance, then the thought experiment can simply be modified to also include a thermal barrier that slows down the dissipating heat but that does not affect magnetic field lines.)

-

“Or do the magnets get more field energy when they are closer together? This doesn't make much sense, and the experiments i have done with magnets don't confirm that. If i have two magnets of a certain strength and i put them together, then i get a magnet with the sum of the strengths of these two magnets. At least this is what i have seen.” – ayeaye

This has been my experience too.

The point is that when demagnetizing one of them the field lines from the other (like with a metal ball bearing) may have a strengthening effect on the internal magnetic alignment and so offer more resistance to demagnetization and so require more energy.

It’s possible.

Which means given them (mainstream Physicists) some wiggle room to avoid the conclusion “the Law of Conservation of Energy is logically flawed.”

If there is a way to craft the argument and eliminate this logical “out” then it makes sense to take it ... it makes sense to use the formulation of the argument (if there is one and there is) that eliminates this “out” (wiggle room).

Yes?  No?

-

“And makes the conservation of energy in that case impossible in any known or unknown way.” – ayeaye


Right.

If you go onto Science Forums and propose and argument that purports to show that the Law of Conservation of Energy is flawed based on “missing or an unaccounted for loss of potential energy” they will simply mock it.

The concept of “potential energy” in mainstream Physics is a bit surreal and they seem fine with that.

And so, again, if there is a way to craft an argument that disproves the Law of Conservation of Energy which does not rely on missing or unaccounted for potential energy (which there is) then this is the argument to use.  The argument linked to in the OP of this thread shows that the two systems end up with different amounts of kinetic energy.  That’s it.  It does not deal with what did or did not happen with the (surreal and ambiguous (in mainstream Physics)) potential energy between them.

(If it seem like I’m arguing against your suggestions, please don’t think so.  In these forums often times people are so wedded to their own ideas that if someone else makes a suggestion and improves on their idea they don’t want to hear it.  It’s just that the point you are raising ... the excellent points and excellent alternative ideas you are raising ... have, often, been the same important points that I worked through and found the “wiggle room” with them on my way to ultimately ending up with the specific argument that I did (linked to in the OP).  So, thank you for all your suggestions.  It’s my hope that you don’t mind it when I say why it think they lead to more logical “outs” than the specific formulation of the argument as it stands.  If you’re not offended, please keep them coming!)



-
 


“Zetetic, Ok, a piece of iron is approaching a magnet, and the magnet shall be demagnetized at different times. Even simpler. I don't think that anyone says that an iron near a magnet makes the magnet anyhow stronger or weaker, and that demagnetizing it takes any different energy, no matter how far the piece of iron is from the magnet.” – ayeaye

I can’t find a link for this.  But, I do believe that, yes, having the magnetic lines of flux pass through the piece of iron rather than just through the air will “make stronger” or “reinforce” the internal magnetic alignment of the magnet.

Again, I can’t find a link for this so it’s just speculation on my part.

The point is, however, it’s possible.  Again, this leads to wiggle room.  And if this (iron in the magnetic field) concept where to be presented to a mainstream Physicists as part of an argument about why “the Law of Conservation of Energy is false” they would take this logical “out” in an instant (whether it is actually true or not).

(Again, I hope it doesn’t seem like I’m arguing for the sake of arguing, or that I’m so wedded to “my” idea that I can’t see other ideas that improve it.)

In the formulation of the argument that I propose this logical “out” is eliminated with “... when the two magnets in the two systems are the same distance apart, the moving magnet in the one system is demagnetized and the fixed in place magnet in the other system is demagnetized ...”.   They can’t wiggle out by claiming “different distances and so different amounts of energy to demagnetize.”


-


“Cool thing you thought out.” - ayeaye

Thank you.  I’ve loved every (failed) pmm design that I have come up with.  And I love everyone else’s attempts.  (While I failed, there is still the hope that someone will find the right mechanical arrangement that I was never able to find!)

This argument against the Law of Conservation of Energy (not a pmm attempt) might also end up as “failed” (perhaps there is a flaw in my logic), but, as of right now, I can’t see one.  Right now, I think I’ve got it.  (But, I’m always open to being shown why I’m wrong.)


-


“If there is any problem that the things there are happening too fast, and the magnet cannot be demagnetized fast enough ...” – ayeaye

In a thought experiment, it is possible to stipulate that the magnet is demagnetized in an instant.

And, like you say, if you want to slow down the heat transfer from the first demagnetized magnet to the second magnet where it too will be demagnetized, then such real world things can be added to the thought experiment.

Thank you for considering my argument!  I appreciate all of the time and effort you have put into considering my idea!

(Again, I hope it doesn’t seem like I’m arguing for the sake of arguing, but rather that I am pointing out why the specific choices I made in crafting the argument where made and why , I believe , were necessary.)


-


“I think that you see now that my analytical thinking is not bad at all.” - ayeaye

I have never thought that your analytical abilities are any less than excellent.

What I do think is that you made a mistake in your use of terminology.  (I think you used “asymmetric” to mean one thing (“omnidirectional”) when talking about magnets and to mean a different thing (“unidirectional”) when talking about gravity.)

To make such a mistake is in no way a reflection of your (or mine (and I make these kinds of mistakes all the time)) analytical abilities.  It’s simply an oversight.

And, yes, the fact that in your last post you have raised as alternative possible arguments many of the alternative possible arguments that I worked though on my way to the actual argument (linked to the OP) that I ended up with, only adds to my esteem of your thinking.

You have no way of knowing all of what I worked through on the way to that that final form of this argument.  And the fact that you are proposing ideas that I also came up with along the way shows that you really have a grasp of the broader scope of the idea!



Again, thank you for continuing this conversation!

Let me know if you have any response to what I’ve just said!



Take care,

- Zet



Title: Re: A possible violation of the Law of Conservation of Energy
Post by: ayeaye on April 28, 2015, 02:22:58 AM
Well, if a piece of iron is in the magnetic field of a magnet, then i think the magnetic field distribution changes, not its strength. Thus when there are no other magnets nearby, it should take the same energy to demagnetize the magnet, no matter what the distribution of its magnetic field is. I can so much say that i have never heard that the strength of a magnet can be changed just by adding iron to it.

But if they can say this, they can say whatever. They can say that something impossible happens, and you cannot prove that it doesn't, because you don't have a complete model of everything that happens there. Everyone has a burden of proof, so when they cannot prove their arguments, they should drop them. But some think that some don't have. That way you can never convince them, even if you are completely right. So maybe you are trying impossible, never do that, and never agree to do that.
Title: Re: A possible violation of the Law of Conservation of Energy
Post by: sm0ky2 on April 28, 2015, 03:33:30 AM
I have a few things to say here, this is an interesting proposal. However, I see some issues that must be resolved before a complete analysis of the systems can be conducted.

1) when examining a magnetic field from two magnetic sources, in terms of potential energy, wherein one or both magnetic sources is free to move towards the other - it must be considered that not all of the potential energy is converted into kinetic energy. Some of the potential energy is converted to kinetic energy, the remainder still exists, though it is not observable from the perspective between the two magnetic sources. This is the total potential energy of the field.
It is distinctly different from the perspective potential energy between the two sources.
       Example: one system the magnetic sources are free to move without restriction.
                      the other system, the moving magnetic sources are attached to a mass that is not great enough to significantly alter the acceleration of the moving magnetic source. In the second example the kinetic energy is greater than in the first, although they both started with the same potential energy. Therefore, there is assumed to be more energy transformation occurring.

2) In the provided example from the author: There is a mass of thermo-chemical mixture attached to the moving magnet in one system, which adds to the kinetic energy of that system. This is not accounted for in the example. (see above)

3) The author states:   "The demagnetized magnet is immediately cut off from the mutual attraction.  But the lack of mutual attraction takes some time to make its way across the distance to the still magnetized magnet.  It remains attracted to the other magnet for a while longer."
     This is not entirely accurate.
 It would be more clear to state that, when the one magnet is demagnetized, the mutual attraction aspect of the field interactions begins to collapse from both ends, and over time dissipates or approaches 0 near or at the center point between the two attracting fields. The influence on the shape of the field of the still magnetized magnet takes some time to realign, but there is not an actual "attraction" during this (very short) time.

The "total energy" of the system is the same in both cases. In the case where the chemical mixture is in motion, more of the initial potential energy has been converted into kinetic energy. But in both cases, in the end, all of the remaining potential energy of the magnetic fields has been converted. Once above the curie temperature, the magnetic domains within each magnet becomes scattered and any remaining magnetic potential energy will have been converted into kinetic energy within the material itself. Domains are no longer aligned in a parallel direction, and the atoms are in an excited state due to heat energy.
On the atomic level, there are many small magnets in rapid motion, in both attractive and replusive interactions.



 
Title: Re: A possible violation of the Law of Conservation of Energy
Post by: JEJEHO on April 28, 2015, 03:16:57 PM
Hi,

Please correct me if I am wrong. In this topic we are comparing two systems, one has low out put and another has high output. Two identical levers with one has high loss due to friction means , one give high out put another give low out put.It didnt violate the Law of conservation of energy.

Likewise in your both system , due to one magnet is fixed its kinetic energy is resticted. so you cannot get the output equal in both systems

If out put is greater than Input then only law of conservation of energy is violated

Regards
Nixon



Regards
Nixon
Title: Re: A possible violation of the Law of Conservation of Energy
Post by: norman6538 on April 28, 2015, 04:11:27 PM
Here is another violation of the law of conservation of energy.
Where did the energy come from that lifts the pendulum past
10 oclock?

the pendulum that is dropped from 2 oclock and rises to 12 oclock and falls back down with no sticky spot.  see that here. http://www.youtube.com/watch?v=4FzK2XKQ-74 (http://www.youtube.com/watch?v=4FzK2XKQ-74)
 
Norman
Title: Re: A possible violation of the Law of Conservation of Energy
Post by: ayeaye on April 28, 2015, 06:07:23 PM
Converting potential energy to heat or to any other form of energy other than kinetic energy, this is a problem. We all have a near infinite potential energy, because we can fall into a black hole somewhere. But our interaction with that black hole is almost nonexistent, and we cannot heat our house with it.

Norman, talk it there http://www.overunity.com/15729/overunity-due-to-asymmetry-of-the-magnetic-field/#.VT-43zr52rM (http://www.overunity.com/15729/overunity-due-to-asymmetry-of-the-magnetic-field/#.VT-43zr52rM) if you think that this is also caused by asymmetry of the magnetic field. We only talked about it here because Zetetic is also interested in magnet motors.

About conservation of energy, i don't know why i don't have any feelings about it. It may or may not be, neither of it disturbs me, but it should make me burn, this looks like wrong.
Title: Re: A possible violation of the Law of Conservation of Energy
Post by: Zetetic on April 29, 2015, 09:52:08 PM

ayeaye,



“Everyone has a burden of proof ...” -ayeaye


This is an interesting point.  And I agree with you in general.  However (... I don’t know what the right word is ...) in “practicality” we have the burden of proof and not them.

If you or I or someone else in this forum claims to have successfully built a perpetual motion machine (or claims to , as I have , come up with a simple two page argument that disproves (to a logical certainty!) a fundamental Law of Physics) I think it is us who has the burden of proof ... or , at least , the greater burden of proof.

When I walk outside and look around the world is obviously flat.  I can see that I’m not standing on a ball.  And at night when I look up into the sky obviously there is a dome over me with little lights affixed to it.  I can see it.  It’s obvious.  And I, and the ground below me, are not in motion.  I can feel it.  It is the Sun that moves across the sky and the moon too.  This is all very obvious.

And so, when fellas like Galileo and Copernicus came along ... you and I can agree that the flat earthers had just as much of a burden of proof as these guys ... but in “practicality” these two had the greater burden of proof.

Everyone back then “knew” the Earth was flat.

And everyone today “knows” that a perpetual motion machine cannot be built and that no one can disprove a fundamental Law of Physics in a simple two page Word for Windows argument using only basic college freshman level Physics concepts.

So ... I agree with you.  I think everyone has, or should have, an equal burden to prove their side ... whether they are on the side of “accepted and known” Physics or whether they are challenging it.

But in practicality, I believe, it is us who carries the greater burden.



 “... i think the magnetic field distribution changes, not its strength.” – ayeaye

I think you are probably right.

I can’t find a link on this that addresses this issue and points to an answer one way or the other.

My point is if there are two ways to fashion an argument, and if both are equally logically sound, but one leads to potentially “unfounded” counterclaims by the other party while the other argument avoids this, then (I believe) it is better to go with the latter argument.

In my everyday life I deal with people who say crazy incorrect stuff all the time.  And if I want to convince them that they are wrong I have to find a way to make my case that counters or avoids their crazy incorrect counterarguments.  Just because I think (and just because I’m right) that what they are saying is wrong doesn’t’ mean that I can convince them they are wrong.



“So maybe you are trying impossible ...” – ayeaye

I hope not.

Maybe it is impossible to show mainstream Physicists that the “Law” of Conservation of Energy is flawed and therefore people working on perpetual motion machines are not just fools who are wasting their time trying to wish water into wine.

I’m going to try (... I may ultimately fail, I have failed a lot in my many years here on this planet ...) but I’m going to keep trying.

 
-


smOky2,


“There is a mass of thermo-chemical mixture attached to the moving magnet in one system, which adds to the kinetic energy of that system. This is not accounted for in the example. (see above)” - smOky2


Right.

In my “casual” description of my argument against the Law of Conservation of Energy I may not have made it clear that the chemical heat packs are at rest.  When the “moving magnet” is demagnetized this is done so when is it near and moving past the chemical heat pack.

If the chemical heat packs where to be moving with the magnet, as you have noted, this would add to the mass.

Here is the link (the same link as in the OP) to the carefully worded description of this argument:  http://www.scienceforums.net/topic/87489-a-magnet-is-demagnetized/page-2



“ It would be more clear to state that, when the one magnet is demagnetized, the mutual attraction aspect of the field interactions begins to collapse from both ends, and over time dissipates or approaches 0 near or at the center point between the two attracting fields. The influence on the shape of the field of the still magnetized magnet takes some time to realign, but there is not an actual "attraction" during this (very short) time.”  - smOky2

I don’t agree with you.

The changes in a magnetic field take time to cross that field.

“... if a giant, huge, powerful magnet appeared one light year away out of nowhere, then it would take exactly one year for magnets on Earth to feel its pull (however small it may be). That is, it would take one year for the "magnetic force" to reach the Earth.”

http://physics.stackexchange.com/questions/5839/does-magnetic-propagation-follow-the-speed-of-light

“... and is defined to be exactly 299,792,458 m/s in vacuum (same as the speed of light).”

http://www.physlink.com/Education/AskExperts/ae445.cfm

“... light is a propagation of electric and magnetic fields. So, if magnetic fields propagated faster than the speed of light, then light would also move faster than the speed of light. That is to say, they don't.”

http://forums.xkcd.com/viewtopic.php?f=18&t=62899



When the one magnet is demagnetized and so the magnetic field goes from one where the two magnets are mutually attracted to one where they are not, this change in the field takes time to make its way across the field and to still magnetized magnet.

This means the still magnetized magnet remains attracted to the now demagnetized magnet for a while longer.  If the loss of mutual attraction where to make its way across the field to the still magnetized magnet instantly, then this would mean that this change would occur faster than the speed of light.  If you are right (and, again, I don’t agree with you) then you have just found a violation of the Special Theory of Relativity and you have disproved this “Law” of Physics.


-


Nixon,


“... one has high loss due to friction ...” - Nixon


You can assume no friction in my thought experiment.  (Whether there is or is not friction, it doesn’t really change the argument.)



-



Norman,

“Where did the energy come from that lifts the pendulum past 10 oclock?” – Norman

You can make a pendulum with gravity and you can make a pendulum with magnets (as you did).

If you make a pendulum with one moveable magnet and with one fixed in place magnet, and if there is no friction, then the moving magnet will end up at the same “height” (the same distance from the fixed magnet) on the other side as it started (two o’clock to ten o’clock).  (See the left side of the drawing below.)

However, if you also have additional attractive magnets on the other side and none on the side where it starts (see the right side of the drawing below) then the moving magnet will reach a greater “height” (two o’clock to twelve o’clock) on the other side.  And there is no violation (sadly) of the Law of Conservation of Energy.  It’s simply that you went from a place with a small amount of magnetic attraction (two o’clock) to a place with more magnetic attraction (twelve o’clock).

Yes?  No?

Have I understood your proposal correctly?



-



ayeaye,

“We all have a near infinite potential energy, because we can fall into a black hole somewhere.” - ayeaye


I agree with what you are saying.

But this is not how mainstream Physics deals with the concept of “potential energy.”

Potential energy, for them, is a matter of designation.  And they typically designate 0 potential energy to be the greatest amount of potential energy possible and so everything else (every lesser amount of potential energy) is a negative quantity.

And so, what a mainstream Physicists would likely say to this comment of yours is that “no there is not an infinite amount of potential energy between you and the black hole but rather 0 amount of potential energy between you and it and as you get closer, and so the amount of potential energy between you and the black hole decreases, the amount of potential energy then becomes – 10 units of potential energy, and then – 100 units of potential energy, and then – 1000 units ... and so on and so on.”

Again, this is not my argument.  It is theirs.

And so, if I want to prove that the Law of Conservation of Energy is flawed and therefore wrong (... which I do ...) then I need to craft an argument that stays away from this stuff (... which I have).


“About conservation of energy, i don't know why i don't have any feelings about it. It may or may not be, neither of it disturbs me, but it should make me burn, this looks like wrong.” - ayeaye


If the Law of Conservation of Energy is true (if the total amount of energy in the Universe is always constant) then every attempt in this forum to build a perpetual motion machine (a device where less energy is put into it and more energy comes out of it) is a total waste of time (including your Field lines chain motor 4/4) and nothing more than regular old non-godlike humans, such as ourselves, trying to wish water into wine.


-


Thank you all (ayeaye, smOky2, Nixon, Norman) for continuing this conversation!




Take care!

- Zet






(PS:  Oops.  I edited this because  I made a mistake and attributed a smOky2 quote to ayeaye.  Oops.  Sorry.)
Title: Re: A possible violation of the Law of Conservation of Energy
Post by: norman6538 on April 30, 2015, 01:21:34 AM


Norman,

“Where did the energy come from that lifts the pendulum past 10 oclock?” – Norman

You can make a pendulum with gravity and you can make a pendulum with magnets (as you did).

If you make a pendulum with one moveable magnet and with one fixed in place magnet, and if there is no friction, then the moving magnet will end up at the same “height” (the same distance from the fixed magnet) on the other side as it started (two o’clock to ten o’clock).  (See the left side of the drawing below.)

However, if you also have additional attractive magnets on the other side and none on the side where it starts (see the right side of the drawing below) then the moving magnet will reach a greater “height” (two o’clock to twelve o’clock) on the other side.  And there is no violation (sadly) of the Law of Conservation of Energy.  It’s simply that you went from a place with a small amount of magnetic attraction (two o’clock) to a place with more magnetic attraction (twelve o’clock).

Yes?  No?

Have I understood your proposal correctly?



No, a normal pendulum never goes past its dropped point.  But this one does. so given
the law of conservation of energy where did the energy come from to make it go
2 extra hours past the dropped point? very simple question.
It demonstrates to me that there is something beyond the law of conservaton of energy.


Norman
Title: Re: A possible violation of the Law of Conservation of Energy
Post by: Zetetic on April 30, 2015, 02:25:24 AM

Norman,



Yes.  I did not address your specific question:  Where does the energy come from to move the pendulum past its “drop point”?



The answer is that there is “potential energy” between the moveable magnet and the magnet at six o’clock and there is also “potential energy” between the moveable magnet and every other magnet in your system.

When the magnet moves from its starting position, two o’clock, to the magnet at six o’clock, the potential energy between the two magnets at the start decreases and its velocity (kinetic energy) increases.  The loss of potential energy is equal to the increase in kinetic energy plus the loss to thermal energy (due to friction).

And then the moveable magnet moves onto the other magnets.  As the moveable magnet moves onto the other magnets and away from the magnet at six o’clock the attraction between the moveable magnet and the six o’clock magnet will tend to slow it down.  And if these were the only two magnets in the system then it would be slowed down and brought to a stop short of ten o’clock.  (It would reach ten o’clock if there was no friction, but since this is a real world model with friction it would stop short.)

However, these are not the only two magnets in your system (if I’ve understood your video correctly).  After the moveable magnet passes the magnet at six o’clock and so is slowed down by moving away from it, the moveable magnet is also moving towards other magnets.  As it move towards these other magnets the potential energy it has with these other magnets is transformed into kinetic energy (and thermal energy due to friction).

And so, while the attraction with the magnet at six o’clock will slow it down as it “rises” on the other side, the attraction with the other magnets will speed it up.  In other words, overall, it will not slow down as much.

And so, since it is deceleration more slowly on the second side than it accelerated on the first side it will reach a point “higher” than where it started (twelve o’clock).

The answer to “where does the energy come from to pass ten o’clock?” is it comes from the potential energy between the moveable magnet and the other magnets on the second side.  This potential energy is converted to kinetic energy and the moveable magnet thus passes ten o’clock.




Think of it this way.  Say you have a magnet taped to the underside of your desk, and on your desk you have a magnet glued to a plastic toy car.  If you set the car and magnet some distance from the taped magnet and then let the car go it will move towards the taped magnet and then past it.  And, if there was no friction it will move the same distance past the taped magnet as from where it started.  Here we have potential energy (at the start) being turned into kinetic energy (which reaches its peak when the two magnets are closest together) and then kinetic energy turned back into potential energy (as the car come to a stop on the other side).  Now, say we did this again, but this time we also tape a second magnet under the desk and on the far side of the first magnet from where the toy car and magnet start.  The toy car and magnet will again move past the first magnet, but his time it will move a greater distance than from where it started.  It will do this because there is potential energy between the magnet on the toy car and both of the magnets taped under the table.  (And if the first magnet taped under the table is stronger than the second magnet taped under the table, then after the toy car moves the greater distance on the far side than from where it started, it will then return to the point closest to the first taped magnet, as it appears is happening with your pendulum in you video.)

The energy to reach twelve o’clock comes from potential energy.  And, at all times, energy is conserved.



Yes?  No?

Am I still not getting your point?

Let me know!




Cheers!

- Zet


Title: Re: A possible violation of the Law of Conservation of Energy
Post by: norman6538 on April 30, 2015, 02:58:25 AM
Zet - you are the only one who has even been curious enough to try an answer. So
congradulations but a few things. Each magnet is the same strength. The positions
are very critical or it will not perform well. The pendulum is not vertical but slanted up about 20 degrees. That matches the gravity force with the magnetic force.
there are magnets on the pendulum that are 90 degrees to the flat magnets from six oclock to midnight.
Then the next big question is why does the magnet go faster clockwise and slower when dropping back down counterclockwise? I think its about the direction of spin in the magnetic field that sorta makes the Lenz counter to motion force when dropping counterclockwise.

Truely the excess energy comes from the magnets. Of all the magnets I have worked
with the attracting force is always less than the holding back force when the magnet
passes center or else the pendulum would speed up as it goes by center.

The only potential energy I see is in the magnets attraction force.
Potential energy is introduced when the pendulum is lifted up to 2 oclock
and that is all used up before it reaches 10 oclock....

One of the unique parts of this setup is several magnets from 6 oclock to midnight.
A little bit of spin added energy from each one and you get 2 extra hours of travel.

Zet - you are very much awake while thousands are still sleeping.

Norman
Title: Re: A possible violation of the Law of Conservation of Energy
Post by: sm0ky2 on April 30, 2015, 03:27:28 AM

No, a normal pendulum never goes past its dropped point.  But this one does. so given
the law of conservation of energy where did the energy come from to make it go
2 extra hours past the dropped point? very simple question.
It demonstrates to me that there is something beyond the law of conservaton of energy.

Norman

The pendulum travels 2 hours past its drop point, then drops from there to where?? slightly below the point it was dropped from?
If this were a true violation, the pendulum would swing higher and higher and eventually make a complete circle.

The magnetism alters the potential of gravity during that part of the swing. i.e. the force of gravity is not acting as much to pull the pendulum back down. There is also a magnetic potential which is behaving in a similar manner.
Thus the pendulum does not actually "gain" any energy.

NOW: if you were to lock the pendulum at this new height, and move the magnets to the other side before releasing it again......hmm
Title: Re: A possible violation of the Law of Conservation of Energy
Post by: sm0ky2 on April 30, 2015, 04:28:18 AM
smOky2,


In my “casual” description of my argument against the Law of Conservation of Energy I may not have made it clear that the chemical heat packs are at rest.  When the “moving magnet” is demagnetized this is done so when is it near and moving past the chemical heat pack.
It takes some time to heat the magnet, as well as some time for the full disassociation of the magnetic domains within the material.



Quote

The changes in a magnetic field take time to cross that field.

“... if a giant, huge, powerful magnet appeared one light year away out of nowhere, then it would take exactly one year for magnets on Earth to feel its pull (however small it may be). That is, it would take one year for the "magnetic force" to reach the Earth.”

“... light is a propagation of electric and magnetic fields. So, if magnetic fields propagated faster than the speed of light, then light would also move faster than the speed of light. That is to say, they don't.”

a magnetic field propagates instantaneously. It is the effects of this field which propagate at the speed of light. i.e. our ability to physically detect the change in flux occur.
If we had a magnetic field, lets say 2 light years across. And we demagnetized or turned off the EMF coil, etc.
The field would begin to collapse at a point 1 light year from the source in every direction. From the outer extremities towards the center of magnetism.






The magnetic field of a photon is spherical, and considerably larger than the photon itself.
Consider a photon at the moment it leaves an electron. The field is already ahead of it. The field arrives before the photon impacts its' target. If the field did not propagate faster than c, it would not be spherical, and it would never be ahead in the path of travel.
Rather, the field would train behind the photon. Which, simply is not the case.

There are ways this has been proven. My favorite example is in Ed Leedskalnins book : Magnetic Currents
Where in, he sets up a demonstration to send electricity through the air.
   This was proven to propagate the magnetic field of an electrical signal, faster than the signal itself. Thus enabling electrons to ride on their own magnetic wave, the same way a photon does.
Title: Re: A possible violation of the Law of Conservation of Energy
Post by: ayeaye on April 30, 2015, 09:45:47 AM
Norman, how does your device really work? You should draw a diagram, for there to be anything to talk about.

Is it just a chain, where every next magnet is further from the center (if it works with attraction, or closer to the center when it works with repulsion)? Such chains are known, and they work because of potential energy as Zetetic said, no overunity.

My field lines chain is different, in that it works based on the asymmetry of the magnetic field, as i have explained. And the experiments show that as well, the additional energy gained there cannot be because of the potential energy, because no further magnets in the chain provide any additional energy potential. All magnets in the chain are the same strength, the same distance away from the center of the disc. The stator magnet every time goes through the magnetic field of every rotor magnet N S, and the N S are equal. The magnetic field is symmetric in that sense, it is asymmetric at every pole.

The only potential energy is the repulsion of the first pole of the first magnet. In the end the stator magnet does not remain locked by any magnet in the chain, but it stops at the position where it is not attracted or repulsed by any magnet (none is greater than friction). Different from the attraction chains based on potential energy, which in the end remain locked. Then it only needs to be given a potential energy of repulsion of the first magnet, to go through the chain again. And it evidently gets more energy by that, than that potential energy.

I have seen a very complex mechanical and magnetic device which was shown to rotate continuously. This may not be a hoax, it might be possible when using the asymmetry of the magnetic field properly, but such device would not be simple because of going through the magnetic field by a proper trajectory. And i think other overunity magnetic devices likely work because of asymmetry of the magnetic field as well, in a way or another.

I say my assumption is this. Given a chain of magnets, each of which is towards the stator magnet with the opposite pole (attracting), and every next magnet is closer to the edge of the disc. The total energy given by such chain is equal to the energy given by the attraction of the last magnet (closest to the stator magnet). Because the energy which any of these magnets give by their positive attraction, they take away by their negative attraction. Because the magnetic field of their poles is completely symmetric.

Now if anyone could show by experiment that the energy which such chain gives is greater than the energy given by the attraction of the last magnet, then this would mean overunity. Unfortunately though the experimenters never measure the energies properly, so this question cannot be answered. When working with repulsion, it's the same, just opposite. If anyone would show experimentally that such "potential energy chain" actually provides more energy than is necessary from releasing it from the lock, then that would be great of course. But so far it should be assumed that it does not.

And when it does, then i see no theoretical reason why that happens, because the fields of the magnets are used in a symmetric way. Then one should also provide a theoretical explanation. Norman mentioned "spin", is that some additional asymmetry of the magnetic field? I'm not aware of that, and it only makes sense to explain it theoretically when any experiment shows that there is any overunity because of that. And knowingly there are no such experiments so far, unless Norman shows that his experiment indeed provides any overunity.

To say all that briefly, i don't think that going through symmetric magnetic fields of no matter how many magnets with no matter what strength, can provide any additional energy. And there is knowingly no experiment which shows that this happens. Unless someone can show otherwise, and show that there is a "spin" or another additional asymmetry of the magnetic field, which makes that possible.

There should be a proper theoretical framework, for all that work to be any research, and anyhow useful. The theoretical base which i propose for permanent magnet devices, is asymmetry of the magnetic field.
Title: Re: A possible violation of the Law of Conservation of Energy
Post by: sm0ky2 on April 30, 2015, 11:31:38 AM
@AyeAye,

There are a few ways this can be set up, but they will all behave in a similar manner. (few exceptions noted)

Basically, the magnetic assist is a linear magnetic gate. the moving magnet enters the field and is pushed/pulled through the array.

if there is a repulsion boundary, momentum carries the moving magnet through the barrier into the gate.

if it is strictly attraction, momentum carries the moving magnet out past the attractive barrier at the end of the gate.

Gravity is countered both by momentum and magnetism in the upwards direction.

Magnetism is countered by momentum and gravity back down through the gate.

in a perfectly symmetrical gate (which is difficult to build, by the way) the gate will actually assist in both directions,
giving acceleration on the way back down as well.

in either case, the effects on the gravitational force are symmetrical in both directions. what you gain on the way up is what you get back out on the way down.
Thus leaving you with exactly what you started with.

The gain in height on one side is only "apparent".
It is possible to place gates on both sides of the pendulum, assuming the magnet doesn't 'hang' at the starting point.
and the pendulum can swing higher on both ends, thus appearing to oscillate with a greater swing distance than a regular pendulum. Understand that gravity is NOT E=mgh during this part of the swing.
The proper analysis will be  E = m(g-T)h Where T is the force imparted by the magnetism. This is not exactly precise, as gravity has a time constraint, where-as the magnetic force does not, so in actuality its a more complex equation.

Before the pendulum reaches position 3, where it would normally stop, it is under the influence of the magnetic gate array.
the magnets "assist" against gravity, lifting it to position 4.
whether it is set up in repulsion, or attraction, doesn't make a great deal of difference in this scenario. The end-result will be about the same.
Title: Re: A possible violation of the Law of Conservation of Energy
Post by: ayeaye on April 30, 2015, 01:42:30 PM
Oh i see, then in Norman's experiment it's just a "magnetic gate array", as sm0ky2 called it, likely an attraction array, because it has no kinetic energy in the position 3. And in the position 4 it will then be locked. Just as i thought, but i called such magnet chain before a "potential energy chain". As i said, such magnetic gate array, no matter whether attractive or repulsive, should not provide any overunity, because the magnetic fields of the poles of the magnets there are symmetric.

Unless someone shows experimentally for example that the energy gained in the attraction chain (attraction gate) is anyhow greater than the energy provided by the attraction of the last magnet in the chain. And that there is some "spin" or whatever in the magnetic field, which makes that possible. Which no one knowingly has done yet.

As i have said, my field lines chain is completely different from the magnetic gate array. There the stator (rotor) magnet is towards the chain magnets by one pole, the same as in the magnetic gate array. But the magnets in the chain don't face the stator (rotor) magnet by pole, the chain is instead as follows, and all magnets are at the same distance from the edge (of the disc):
           
          | N | --->
          ===
[N S]   [N S]   [N S]   [N S]   [N S]   [N S]   [N S]   [N S]

I think potential energy is not real energy. It is a potential to get energy in some field. Yes the field does work. If we have a large magnet, and we let an iron ball to fall on it, then we get energy, first as a kinetic energy, which is then transformed into heat. And if we have a large number of such iron balls, then we can get a lot of energy. But when we want to get any of the balls back from there, then we have to use as much energy as we got, when the ball fell to the magnet.

So one may call it overunity, less energy in than we get out, indeed. But in overunity we have to consider the potential energy, otherwise overunity just doesn't make sense. Because the only reason we need overunity, is for continuous work. Otherwise one can do an experiment, let an object to drop from one's hand, and say overunity. No need for other experiments then, overunity is proven. This is the trick used in some overunity experiments, they say overunity, but they define overunity wrongly, so there really is no overunity.

So the overunity is defined like that, considering the potential energy. Because of its purpose, to repeatedly get energy. We don't need a continuous work in the experiments though, and its better when the device doesn't work continuously. Because it is likely impossible to measure the energies then. But we have to show by measurements, that there is overunity the way the overunity is defined.

No one is interested in any benefit of getting energy though, the only purpose of overunity experiments is theory. Using anything practically is primitive, the higher benefit is the theoretical thinking, and the creativity involved in that. I explain. We put all our efforts to do the theoretical work. We get no money for that, and we let others to take from us money. So we are in agony and we die in agony. And we get joy out of that agony.

That iron ball, we need the same energy to get it out from the magnet, than we got when it fell to the magnet, because the field is symmetric. It is not because of the mystery of potential energy. In the symmetric field, when moving something to somewhere, moving something back from there, takes the same amount of energy as we got when moving the thing to there. No matter by what trajectory we moved the thing, in one direction or the other. The same is not always true in an asymmetric field.

So in a symmetric field, the field can do work, every field can do work. But symmetric field cannot do continuous work. Thus for overunity we need an asymmetric field, so that the energy we get is greater than the initial potential energy in the field.
Title: Re: A possible violation of the Law of Conservation of Energy
Post by: norman6538 on May 01, 2015, 01:44:51 AM
Norman, how does your device really work? You should draw a diagram, for there to be anything to talk about.

Is it just a chain, where every next magnet is further from the center (if it works with attraction, or closer to the center when it works with repulsion)? Such chains are known, and they work because of potential energy as Zetetic said, no overunity.


ayeaye
If I really knew how it worked I'd draw it. The magnets are pretty symetrical not
closer closer.  As I recall if you position the magnet near any magnet it will just
get stuck there.  So you can see its a mystery to me....

Norman
Title: Re: A possible violation of the Law of Conservation of Energy
Post by: sm0ky2 on May 01, 2015, 01:53:37 AM
its actually a lot harder than one may think to make the field of such arrays perfectly symmetrical.
you can measure down to the micrometer, but the field responds to the nanometer. perfect machining comes real close, but the average build at home project, not so much....

asymmetry does not matter in most situations. The asymmetrical field is still inversely proportional. +/-, it all balances out in the end.
Title: Re: A possible violation of the Law of Conservation of Energy
Post by: ayeaye on May 01, 2015, 07:08:58 PM
The asymmetrical field is still inversely proportional. +/-, it all balances out in the end.
No. The poles are inverse, yes, but they are inverse in a way that when the asymmetry at one pole gives additional energy, the asymmetry at the other pole gives additional energy too. Because of symmetry between the poles, they don't compensate each other in that respect, but both add to the effect. It is a symmetry, the same which is true for repulsion at one pole, is true for the attraction at the other pole. Like less repulsion when entering and more repulsion when exiting at one pole, usually means more attraction when entering, and less attraction when exiting, at the other pole, in case of asymmetry such as in my field lines chain. And this is exactly because of symmetry between poles.

This asymmetry of poles is caused by more field lines between the poles, less between the magnets, and these between the magnets somewhat directed away. Not a great asymmetry, but there is some asymmetry, and by experiments this seems to be enough to provide some overunity, not much overunity. Because of that arrangement of magnets, the field is symmetric between the poles, and this adds to the effect.

But this is yes a very subtle thing, the magnets have to be placed rightly, and be of right strength, for field to be asymmetric. Most importantly the magnets have to be weak, so that there is not too much interaction between them. The stator magnet at that can be stronger. It's so in my experiment, but there may be other arrangements.

When going only through one pole, there may even be a piece of iron instead of magnet. This complex continuously rotating magnet device that i saw, had iron balls going through the magnetic field of a magnet, by some weird trajectory. I think this was using the asymmetry of the magnetic field as well. The device rotated continuously, but could do only that, it was not powerful enough to extract any energy from that.

Norman,

Yes, the rotor magnet can get stuck at every magnet of the attraction gate array. It will not if it goes over them with speed, but finally it will be stuck at the last magnet of the array.

But i don't know what your device is. It looks like attraction gate array, but then it does not stick to the end position, which is weird. It may be overunity, but without knowing what it is, this cannot be decided either. Because it can be whatever, like some spring inside the rotor can do the thing, even when there are magnets. A complete data about a thing like this has to be available, so that it can be replicated. Otherwise even if it is something great, it is worthless as an experiment.

Array, i often call it chain. But i also mean something deeper by chain, an array of magnets, and how their poles are connected with field lines. This, magnetic gate array, doesn't form though what i call the "field lines chain". Because the magnets in that array repulse each other, and field lines don't form a chain.
Title: Re: A possible violation of the Law of Conservation of Energy
Post by: ayeaye on May 02, 2015, 11:21:29 AM
Norman,

Your experiment nevertheless made me to think. Maybe overunity should be defined without potential energy after all. Because potential energy, by everything it implies, is so confusing concept. Potential energy is not the same as potential to get energy, because the latter implies less. But potential energy implies things which have no meaning in every asymmetric field, at least not in every imaginable asymmetric field. So then, overunity could be defined as less energy in, more energy out, so that it can be repeated. In case of dropping iron balls to magnet this is not true, because iron balls are a limited resource, and we eventually run out of it.

I mean i thought, if it's really an attractive magnetic gate array. Which is made to be minimal, so that it works only when the rotor has some speed. And then it doesn't stick in the end because the gravity pulls it back. Does it really work? I have not tried it. If it really is that and it does work, then it's overunity, because there is a process with more energy out than energy in, and this process can be repeated. And again yes, not enough for continuous rotating, or continuous swinging there i guess. But this is not a necessary criterion for overunity.

So when this device really works like that, then it works by a slightly different principle than my experiment. I looked at only one field, and when there is only one field, then for overunity it has to be irregular. That is asymmetric, shielded, or possible to change or switch off by small energy. The last two as much as i know are not true for magnets, so only asymmetry remains. Though i'm not entirely sure. So my device works because of asymmetry of the magnetic field.

But your device, if it is what i guessed it may be, works differently. Because there are two fields, magnetic field and gravity, working at the same time. Both i assume are symmetric. The gravity field and all the magnetic fields there are symmetric, that is. So two different symmetric fields may also provide overunity. Because they may provide the irregularity which a single symmetric field cannot provide. So the potential energy concept may be entirely true only in a single symmetric field. This is interesting, if true.
Title: Re: A possible violation of the Law of Conservation of Energy
Post by: sm0ky2 on May 02, 2015, 12:29:59 PM
No. The poles are inverse, yes, but they are inverse in a way that when the asymmetry at one pole gives additional energy, the asymmetry at the other pole gives additional energy too. Because of symmetry between the poles, they don't compensate each other in that respect, but both add to the effect. It is a symmetry, the same which is true for repulsion at one pole, is true for the attraction at the other pole. Like less repulsion when entering and more repulsion when exiting at one pole, usually means more attraction when entering, and less attraction when exiting, at the other pole, in case of asymmetry such as in my field lines chain. And this is exactly because of symmetry between poles.

This asymmetry of poles is caused by more field lines between the poles, less between the magnets, and these between the magnets somewhat directed away. Not a great asymmetry, but there is some asymmetry, and by experiments this seems to be enough to provide some overunity, not much overunity. Because of that arrangement of magnets, the field is symmetric between the poles, and this adds to the effect.

But this is yes a very subtle thing, the magnets have to be placed rightly, and be of right strength, for field to be asymmetric. Most importantly the magnets have to be weak, so that there is not too much interaction between them. The stator magnet at that can be stronger. It's so in my experiment, but there may be other arrangements.

When going only through one pole, there may even be a piece of iron instead of magnet. This complex continuously rotating magnet device that i saw, had iron balls going through the magnetic field of a magnet, by some weird trajectory. I think this was using the asymmetry of the magnetic field as well. The device rotated continuously, but could do only that, it was not powerful enough to extract any energy from that.

There are always the same number of "lines", what goes in, must come out. Every N needs a S to travel around its' loop.
 In an asymmetrical configuration, they are closer together on one end, more spread out on the other. essentially the strength (Gauss/Tesla) can be measured stronger at one pole vs the other. This is a matter of "lines" of flux per spatial volume. 

Weaker magnets, such as ceramic-ferrite, are more pliable. They are easier to warp the fields into the shape you want. A strong magnet like neodymium holds its' shape more in resistance to manipulation. This is caused by a property of magnetism that causes a minimum distance between "lines" when the field is compressed. The stronger the magnet, the larger this minimum distance.
For instance, a rubber refrigerator magnet can be compressed entirely into the magnetic material. Leaving no apparent field on one side, and an extended field on the other. The "lines" are still there, they are just compressed to the surface of the magnet.

Angle of entry into the field is of great importance. Some magneticians will tell you that potential energy at a given point is the same, regardless of the path taken to get there. I disagree with this, as it only really applies to a symmetrical fields (and not always). In certain configurations, there can be a path of least resistance where-in a larger potential is observed via a different exit route.

[ I use the term lines in " ", because I'm not sure this is the best way to describe it. It's convenient as a way of visualizing the field, but I like to think of it more as a bunch of Nascar tracks, with millions of tiny pulses traveling around at some ridiculous  velocity.]

Potential Energy::  When you examine a system from the perspective of energy potentials, you must consider where this potential exists. If it exists outside the boundaries of your system, then input / output energy is not the same at the potential.
They cannot be compared in that manner for the purposes of OU or COP.
Often times, a subject is argued from the perspective of E=mgh or some other analysis of a potential, without consideration that the gravitational field exists well outside the boundaries of the system observed. Energy passing into the system is not taken into account, and therefore the system is assumed to be OU. When in all actuality, the energy can be derived from outside the system boundary. Magnetism exists in a similar manner, but without a time constraint. A magnetic field extends infinitely across the universe, with ever decreasing magnitude. So while your system might be the area around your workbench, the magnetic field encompasses everything outside of that as well. So, potential energy, while it is a good method of analyzing energy values, caution must be taken as to how it is used to define your system. It is not equivalent to the input and output energy.
Especially when you consider that potential energy of a larger field can be manipulated in a localized area within that field.
like a vertical wind tunnel suspending an object in air. or magnetic repulsion, buoyancy, etc..

Here's something to think about. If you fly an airplane up to 9000 feet above sea level, then travel horizontally to some other location, maintaining the same altitude, in terms of gravitational potential, the energy is the same? or is it?......
Let's say you travel from Vancouver to Miami. In Vancouver the energy potential is: E = m[g-(change in g)]h
in Miami, where gravity is closer to the average, E=mgh.
Is this overunity?  No, the energy comes from the gravitational field outside the system being analyzed. Earth density is greater in Miami, thus a greater potential energy at the same height.
This sort of thing can also happen with magnetism.
Title: Re: A possible violation of the Law of Conservation of Energy
Post by: ayeaye on May 02, 2015, 05:55:29 PM
Norman,

Well, even considering the potential energy, in the magnetic gate array, when the field is symmetric, and the potential energy makes sense, your device still has overunity. When it really is what is on that diagram drawn by sm0ky2. Because in addition it moves back from the end position, to position 2, by gravity, the energy of that movement is not provided by any initial potential energy.

Zetetic,

I think the answer to your question is, in the irregular field (like that which can be switched off) potential energy has no meaning. Comes out of logic. One may analyze what happens in the magnetic field, if there is any way to do that. But, that magnetic field increases, when an iron ball approaches, the problem with that is that this may violate the conservation of energy. Because the potential energy of the iron ball is transformed into a kinetic energy exactly by the law of the conservation of energy. If there in addition is any transfer of energy to increase or decrease the magnetic field energy, then this as i see violates the conservation of energy. I mean, i don't exclude that some of the energy of the moving iron ball goes to the magnetic field, but i mean that he magnetic field increasing or decreasing by the amount of the kinetic energy of the iron ball, violates the conservation of energy. Or that the energy spent to remove a slightly greater magnetic field takes much more energy, this should be exactly equal to the field energy of the magnetic field. Each magnetic field has a field energy, but this energy is very small.

sm0ky2,

Yes a pole has the same number of field lines, but they are distributed differently. Say when outside is another attracting magnet, a part of the field lines at the outer side of the pole goes to the pole of that magnet, and less remain free. To the point when two magnets are against each other by opposite poles, we cannot detect any poles from outside at that point any more, two magnets have become one magnet. Then we cannot use the asymmetry of that pole any more, because the pole like disappeared, the outer poles of the joint magnet work instead. At least this is how i see it by now by my simple mind.
Title: Re: A possible violation of the Law of Conservation of Energy
Post by: Zetetic on May 03, 2015, 01:19:53 AM

smOkey2,


“It takes some time to heat the magnet, as well as some time for the full disassociation of the magnetic domains within the material.” – Reply #61

For the ease of analysis, assume (and this is acceptable to do in a thought experiment) the demagnetization is instantaneous.  (If you assume something else, a slower more real world process, it does not change the overall logic but rather just makes the thought experiment more cumbersome.)


-


“The field would begin to collapse at a point 1 light year from the source in every direction. From the outer extremities towards the center of magnetism.” – Reply #61

smOkey2 please respond to this question:

Are you saying that if we have an electromagnet and a permanent magnet 1 light year apart , ... and where the electromagnet has been turned on for a number of years and the permanent magnet and the electromagnet are attracted to one another ... and so the two magnets feel a pull towards one another, ... are you saying that if that electromagnet is then switched off that the permanent magnet will feel the loss of the pull from the (now switched off electromagnet) in less than one year (from the time when the electromagnet was switched off)?



---


Norman,

Okay, I understand now that the pendulum comes to rest at the six o’clock position because this is the point closest to the Earth below and not due to greater magnetic attraction.

However, the analysis basically remains the same.  When the pendulum is at two o’clock there is potential energy between it and the Earth below (due to gravitational attraction) and there is potential energy between it and the magnets on the far side (due to magnetic attraction).

I have spent many many years working with models like these (with the magnets orientated towards one another in all sorts of different ways) trying to get OU.  And I failed.  (That’s not to say that someone else can’t find that just right configuration that I was never able to find.)

My experiences have lead me to the same conclusions as stated by sm0key2:

“what you gain on the way up is what you get back out on the way down.” – Reply #63

“asymmetry does not matter in most situations. The asymmetrical field is still inversely proportional. +/-, it all balances out in the end.
” – Reply #66


But there is always hope!



---



ayeaye,


“I think the answer to your question is, in the irregular field (like that which can be switched off) potential energy has no meaning.” – Reply #70

“Maybe overunity should be defined without potential energy after all. Because potential energy, by everything it implies, is so confusing concept.” – Reply #68


I think “potential energy” should be avoided (if able) when making purely theoretical arguments as against the “known and accepted” Law of Physics (such as mine linked to in the OP) because it is such a slippery concept, but when it comes to building OU mechanical devices it should be fine.

When you’re building a device that is intended to move forever, and if that devices includes some form of energy becoming potential energy and then that potential energy becoming another form of energy and so on, what you’re dealing with is a real thing.  When two magnets are some distance apart from one another, there really is an attraction between them, and if let go this potential energy will really become kinetic energy.  Potential energy in this context is not simply a matter of designation.  And if anyone wants to designate it as 0 units of potential energy or -10 units of potential energy or a zillion units of positive potential energy ... it doesn’t matter.  In your OU device that real potential energy becomes real kinetic energy.

However, in the realm of a purely theoretical argument, if you were to say “when you demagnetize one magnet in a two magnet system there is a violation of the Law of Conservation of Energy because the amount of potential energy between the two magnets due to their mutual attraction is now gone without a corresponding change in another form of energy to offset this” then that’s, I’ve found, where the dance of designation occurs and mainstream Physics gets slippery.


-


I think you and sm0key2 are in danger doing what you and I did and talking about the word “asymmetrical” with two different concepts in mind.  And, if so, I hope you two can avoid my mistake of then going on for long post after long post telling you that you were wrong, when all along I just didn’t understand how you were using that word in relation to magnetism and in relation of gravity.  My bad.


I want to understand your 4.4, and so I’ve made a drawing for you to critique.  Please examine it and let me know how this does and/or does not correspond to your ideas.


---



This has been a fascinating discussion!

Take care!

- Zet



Title: Re: A possible violation of the Law of Conservation of Energy
Post by: ayeaye on May 03, 2015, 03:26:33 AM
Zetetic,

Yes correct. Sure there is some negative repulsion and attraction between the magnets, but this is less than the positive repulsion and attraction near the side of the magnet, due to asymmetry of the magnetic field at each pole. Maybe you should also draw the curvature of the disc, which may provide more asymmetry. I also talked about 3/4, which i think is a better example than 4/4, but i did not have enough weak magnets for a complete circle.

So there were better results when magnets were more far apart than in 4/4. The problem with 4/4 was that these magnets were too strong, and attracted to each other, so the only solution i found was to put some folded paper between them. I think using weaker magnets there should give better results, because all the problems there were caused by magnets too much interacting with each other. Including maybe that shifting of the poles.

I think it's better to define asymmetry so that, when we draw a plane through the center of the field, then the field is asymmetric when there is anywhere more field lines at one side of that plane, than at the other side. This includes asymmetry also when it is only due to uneven distribution of the field lines. In the magnetic field, this center of the field is the center of one pole, so it is the asymmetry of the field around one pole.

I didn't define it clearly enough before also, and this also could cause confusion, this part was my fault.
Title: Re: A possible violation of the Law of Conservation of Energy
Post by: Zetetic on May 03, 2015, 04:24:26 AM



ayeaye,


Okay ... let’s see if I can another step forward (without taking two steps back) in understanding your idea.

I’ve drawn another picture.  Please let me know if I’m on the right track, or not.

(Please note, in this drawing the moving magnet and fixed magnets are opposite from how they are in your actual models.  It was easier to draw it this way.  But, as we talked about before, it doesn’t which one(s) is in motion and which one(s) at rest, but just that their orientations towards one another is the same as in your models.)

(Let’s see if we can get this right first, before moving onto other drawings.)


Let me know!


Cheers!


- Zet



Title: Re: A possible violation of the Law of Conservation of Energy
Post by: ayeaye on May 03, 2015, 05:26:50 AM
Zetetic,

Yes right.

I tried to draw too, but i don't know how well i can draw these field lines.

This is a kind of subtle though. In that the more there is interaction with the other magnets, the weaker the poles come. Until all is one magnet, and poles are only at the ends of the chain.

Where do they sell these rubberized magnets you talked about, btw? I searched ebay, and all i found was magnet tape. It may be possible to make these magnets from that too. Magnet tape should have one pole on one side of the tape, and the other on the other side, so the edge should be N S, but not sure whether that works.

Very small neodymium cube magnets, like 1 mm, may be another option, but some say that neodymium magnets have somehow a very rigid magnetic field, so may not be good for the purpose. Yet another option may be to magnetize tiny ferrite cores or such.

Also it looks like, when the circle of magnets in 4/4 is full, something happens. Partly all the chain behaves like one magnet, and wants to have poles at the ends. But when the circle is full, the distribution of the field kind of changes, with new stronger poles emerging somewhere. And this is more difficult to control, It may sound crazy but, it may be good to omit one or two magnets in the circle, to make a "gap".

Then at the ends of the chain there will be stronger poles, but when the chain is made well, these would not be strong, and the strength of the magnetic field is more evenly distributed around the circle. So maybe it will get enough speed to jump over the gap, not sure though. 3/4 is a simpler experiment, just to show overunity. Trying a continuous rotation causes additional problems, and may not be possible to achieve because of that.
Title: Re: A possible violation of the Law of Conservation of Energy
Post by: Zetetic on May 03, 2015, 10:40:16 PM


ayeaye,


I know you’re eager to get to the curved aspect of your design.  But, first, I need to understand the basics of your idea.

When you have a row of magnets attractively lined up pole to pole, then the lines of flux both go from one pole on one magnet to the opposite pole on the next magnet, and also they loop around to their own opposite pole (see drawing 1 below).

Yes?  No?


And if we draw a line through those field lines (drawing 2) we see that above that line the field lines are “asymmetrical” (drawing 3).

In this part of the field the north to south field lines span across the length of each magnet (3.1) differently (“asymmetrically”) from how the north to south field lines span across the gap between each magnet (3.2).

Yes?  No?


And so if there were another magnet external to this row of magnets (drawing 4), and orientated so that it will be pushed and pulled leftward over the span of each magnet and it will be pushed and pulled rightward over the gap between each set of magnets, then it will not be (this is your contention) pushed and pulled leftward and rightward between these two spans with the same force due to this “asymmetry.”

And so, it will achieve an overall movement in one direction (leftward).  Again, this is your contention.

Yes?  No?



Did I just take two steps back in my understanding of your proposal?  Please let me know!

Take care,

- Zet



(PS:  Your drawings in Reply #74, I believe, have helped me to better understand where you are going with this.  So, thank you for that.  But I’m not ready to comment on that part of your ideas yet.  We need to make sure I understand the basics of your idea first!)


Title: Re: A possible violation of the Law of Conservation of Energy
Post by: ayeaye on May 03, 2015, 11:55:43 PM
Zetetic,

The first two are yes. The third, well, the magnets will get pushed and pulled to the right when over the gap, yes. The assumption is that the speed of the disc enables to go over the gap. The additional energy got by pushing and pulling in the right direction, increases the speed of the disc, and in the opposite direction decreases the speed. But when the speed increases at the side of a magnet more than it decreases at a gap, then the disc gets additional speed, that is additional energy.

This doesn't work without the disc, which has a certain inertia, this enables to gather energy, and go over the gaps.

There are three reasons why there is asymmetry, as i see by now, maybe not all entirely correct. First there is a ferrite, or other magnet material between poles of the magnet, and not between the magnet. This enables a stronger pull between the poles of the magnet. Second, every magnet interacts with the magnet near it, this takes away some field lines, leaving fewer available to go to the stator magnet, when at the gap. Third, because of the curvature of the disc, the nearby magnets are under some angle, taking field lines further away from the stator magnet, at the gaps.
Title: Re: A possible violation of the Law of Conservation of Energy
Post by: Zetetic on May 04, 2015, 12:21:35 AM


ayeaye,


“This doesn't work without the disc, which has a certain inertia, this enables to gather energy, and go over the gaps.” – Reply #76


Okay ... good ... I’m glad we slowed it down.


Are you saying that there is a difference whether the row of magnets is in a straight line or if they are in a circle?

If we have an infinite row of magnets and they are in motion (they have inertia) are you saying that this (the dynamics of the interacting magnets) will work differently than if we have a circle of magnets and they are in motion (they also have intertia)?


Thank you,

- Zet



Edit/

(PS:  I think you may have modified your Reply #76 while I was responding to it.  I didn't see the last paragraph when I responded to it here.  That changes things.)

/Edit


Title: Re: A possible violation of the Law of Conservation of Energy
Post by: ayeaye on May 04, 2015, 12:44:27 AM
Zetetic,

> Are you saying that there is a difference whether the row of magnets is in a straight line or if they are in a circle?

> If we have an infinite row of magnets and they are in motion (they have inertia) are you saying that this (the dynamics of the interacting magnets) will work differently than if we have a circle of magnets and they are in motion (they also have intertia)?

Yes i think there is a difference and a circle is somewhat better, but not that there is no asymmetry in a straight line.
Title: Re: A possible violation of the Law of Conservation of Energy
Post by: Zetetic on May 04, 2015, 01:08:53 AM



ayeaye,


“Yes i think there is a difference and a circle is somewhat better, but not that there is no asymmetry in a straight line.” – Reply # 78


Okay.

So, now based on your last paragraph in Reply # 76 and based on the lines in the drawings in Reply # 73 I can move on and think about what you are claiming (or suggesting) is the dynamic between the magnets (and specifically, the dynamic that leads to OU).

Cool.


- Zet


Title: Re: A possible violation of the Law of Conservation of Energy
Post by: ayeaye on May 04, 2015, 03:31:37 AM
Yes i tried to tell everything i know. Any questions about my experiment, i will answer. Next may be modeling the magnetic fields, but unfortunately i don't know any open source software for this. If someone knows any, please say.
Title: Re: A possible violation of the Law of Conservation of Energy
Post by: Zetetic on May 04, 2015, 06:06:06 AM


ayeaye,



If I understand you correctly ... and , unfortunately , I cannot draw a picture for this ( ... it is beyond my simple microsoft paint skills , to do so ...) ... you are suggesting that because the field lines that come out of the north end of one of the magnets in the row , and some end up going to the south pole of the next magnet in the row ... while the other’s go up and back over to its own opposite pole ... that the push back (the unwanted ( ... “unwanted” in the sense ( let me define my terms ) of bad for this OU design ; they push back the overall motion and slow it down ... ) push back from the north end of the bar in the row of magnets on the external magnet is a lesser force than the push forward from this same pole once the external magnet is beyond to the other side of its repulsive force ; since the magnetic flux lines here ...

And I think that’s where you lose me.

I’ll have to think about it somemore.

Thx.

- Zet



Title: Re: A possible violation of the Law of Conservation of Energy
Post by: ayeaye on May 04, 2015, 09:35:59 AM
Zetetic,

> push back from the north end of the bar in the row of magnets on the external magnet is a lesser force than the push forward from this same pole once the external magnet is beyond to the other side of its repulsive force ; since the magnetic flux lines here

Yes.

But i didn't quite understand the first part of your question.

As much as i understand, at the center of a pole, there are always the same number of field lines for a particular magnet, and the field lines are always distributed evenly. So it is like as if at the center of a pole there were an entirely symmetric field. But then it depends on where all these field lines go. But this is as much as i understand, there may be something i don't know.

Can you simply draw freehand lines in microsoft paint? One of my first drawings here was done in gimp that way. Not difficult, but then i found that drawing on a paper, and then capturing the image with the web cam, is still simpler.

And, if you ever try it, you may think about using multiple stator magnets. But, multiple stators don't give anything there, or i cannot think about any benefit of them. Why? Because that device is dynamic, not static. It is about adding additional speed, and a single stator does it, the same as multiple stators. The energy is added as an additional speed, it is not static. Just have to say everything.
Title: Re: A possible violation of the Law of Conservation of Energy
Post by: Zetetic on May 05, 2015, 01:15:54 AM

ayeaye,



Maybe you’re saying something different.  Let me try this tangent, and see if I can get at your idea.



Please see the following drawing.


If a magnet is alone (top part of drawing), then all of the field lines coming out of the one end of the magnet are matched by an equal number of field lines going into the other end of the lone magnet.

However, if you have two magnets side by side , and magnetically aligned (lower part of the drawing) , then some of the field lines will come out of the one end of the magnet and loop back around to the other end of that same magnet , while some of the field lines coming out of that end of the magnet will not loop around but rather go into the opposite pole of the other magnet.

And so , looking at it from this perspective , it might seem as if there are more field lines coming out of one end of a given magnet and less field lines entering the other side of that same magnet.

Is this what you are getting at?

Yes?  No?  Something else?



- Zet



Title: Re: A possible violation of the Law of Conservation of Energy
Post by: ayeaye on May 05, 2015, 04:47:55 AM
Zetetic,

Your drawing is not entirely correct. When nothing is near a magnet, some field lines also go away from it, both from the south pole and north pole. Otherwise that magnet cannot attract anything from right or left. Magnet attracts only when its field lines cross the object.

There is i think always the same number of field lines at both ends of the magnet. The only difference is how they distribute, how many go to other magnet, and how many just go out. And this, and the direction of the field lines, may provide asymmetry at some place at the pole of the magnet.
Title: Re: A possible violation of the Law of Conservation of Energy
Post by: verpies on May 05, 2015, 12:44:40 PM
There is i think always the same number of field lines at both ends of the magnet. The only difference is how they distribute,
Why do you think so?
BTW: The number of these lines is commonly called the total Magnetic Flux.
Title: Re: A possible violation of the Law of Conservation of Energy
Post by: ayeaye on May 05, 2015, 01:08:59 PM
Why do you think so?
I don't know, i think this is what field lines are. Field lines don't exist by themselves, they are just vector paths of the curvature of the field. Assuming that the field is evenly distributed at the center of the pole, this is how it should be. Why i think the field is evenly distributed at the center of the pole, simply because the field should be the strongest there, and the least affected by outside forces. So this is more or less how it should be. But i don't say that it's precisely so.

No, magnet tape may not be good for the purpose, as it may not have all one pole at one side. Not sure though, and i don't know where to get a better magnetic rubber. So the other options are the 1 mm neodymium cube magnets, or magnetizing small ferrite cores, the latter may be the best option.
Title: Re: A possible violation of the Law of Conservation of Energy
Post by: ayeaye on May 06, 2015, 01:10:53 PM
Well, i could repeat the experiment 3/4, with a chain (array) of four rectangular 7 x 7 x 5 mm ceramic magnets, when the stator was quite far, and the forces were not great. The distance between magnets was such that 16 magnets make a full circle, with that distance between magnets.

7 x 7 x 5 mm ceramic magnets are the smallest ceramic magnets one can buy. The original experiment was done with 5 x 5 x 3 mm ceramic magnets, but they don't sell so small ceramic magnets anywhere. The stator magnet used in the original experiment 3/4 was a cylindrical neodymium magnet 15 mm long and 8 mm in diameter. An easy way to fix a stator magnet, is to put two cigarette lighters one on another, and fasten with a mounting tape.

I also didn't find any shifting of poles when 16 such magnets (7 x 7 x 5 mm) made a full circle. With such distance also the forces between magnets were not too great, and it was easy to fasten the magnets with a mounting tape. What i felt with such distance between magnets of that strength, was that the movement didn't feel so smooth and not so good as with a smaller ceramic magnets.

I couldn't get any continuous rotation with a full circle of 16 magnets on the disc though. The rotation was in fact worse than in the experiment 4/4. But continuous rotation was not my aim, the only aim was to show overunity. If one wants to achieve a continuous rotation in that way, then this is a whole separate work, and i have no reasons to say that it can succeed.

One thing about multiple stator magnets that i mentioned, as i said they make no sense generally, like for going over a gap, because the device is dynamic. Yet they may make some sense because in that design everything may matter, the placement of magnets, but also the strength and distance of the stator magnet. So multiple stators may make sense when that enables to use stator magnets of a proper strength. But they also may not be necessary.

So i think i now said everything i could.
Title: Re: A possible violation of the Law of Conservation of Energy
Post by: Zetetic on May 06, 2015, 10:47:02 PM



ayeaye,


Please excuse my silence.

I can think through things in incredible detail.  And, so, if you’ve got something with your Fields Lines Motor, I may be the right guy to understand you.

However, I burn out.  I need to take a break from it for a while.  I will come back.  But I need to think about other things for a time.

Here is what I am debating (/thinking about) at the moment:

http://www.scienceforums.net/topic/88857-what-happens-at-a-distance-when-an-electromagnet-is-turned-off/

Please note: I use the word “asymmetrical” in that argument a lot.  And I got that from you!

In this argument (linked to here) I think I have disproved Newton’s Third Law.  (I just stumbled across it.)  And, if I’m right, then I can disprove it in only on short paragraph.

(In the link in the OP of this post I disprove (hopefully) the Law of Conservation of Energy.  And I do this in only two (Word for Window) pages.  To get it into that short of an argument is an accomplishment.  However, to disprove another fundamental “Law of Physics” (Newton’s Third Law) and to do it in only one clear simple short paragraph is an even greater accomplishment.)

So, anyway, I’ll be back (here).

But, right now, I need to think about other things.

(I’m also taking a Coursera course on Soren Kierkegaard.  Professor Jon Stewart is an excellent teacher.  You might not think that Kierkegaard has anything to do with what we are talking about here, but upon a deeper reading of what Professor Stewart is saying and then upon a reflection of our conversation here, there is a lot of relevance!)

So, please excuse my ( ... temporary ... ) silence.  My brain cells need to rebuild.



Thank you for the conversation thus far!  I have enjoyed it!  And, as always, I respect your logic and especially your creativity with that sound logic immensely!



Cheeers!

- Zet


Title: Re: A possible violation of the Law of Conservation of Energy
Post by: verpies on May 11, 2015, 08:10:42 AM
I don't know, I think this is what field lines are. Field lines don't exist by themselves...
Regardless whether these lines are real or imaginary their amount is not constant in a permanent magnet and depends on the reluctance of its external magnetic circuit.  This is evident from the work line of the BH curve.

However, an energized superconducting tube keeps the total flux through the tube constant regardless of the external reluctance and it's flux distribution resembles the distribution of a permanent bar magnet.
Title: Re: A possible violation of the Law of Conservation of Energy
Post by: ayeaye on May 11, 2015, 06:00:00 PM
Regardless whether these lines are real or imaginary their amount is not constant in a permanent magnet and depends on the reluctance of its external magnetic circuit.
Yes right. So i assumed that the reluctance doesn't change significantly. When another magnet comes near the magnet, then this certainly decreases reluctance. But this effect should be proportional to how many field lines cross the external magnet, so this likely still does not change in essence that which was said. It is difficult to draw that increase of magnetic field, especially if it's small, has to be a very detail drawing. I would like to have some software that models magnetic fields, but again i have not found any of such open source.

Thinking about this canceling the magnetic field, which Zetetic talked about. So when a piece of iron comes near the magnet, then this increases the magnetic field? Assume so, but then the energy of the magnetic field is negative? If true, when the magnetic field is canceled when the piece of iron is nearer the magnet, then this takes less energy, because the energy of the magnetic field shall be released. So there is still no way how the potential energy will be converted. Generally that is, when not considering the amounts of energy. The energy of the magnetic field is very small, yet i'm not sure how great it is compared to the kinetic energy. Just what i can think with my simple mind.

Thank you Zetetic, i think you are great. We are here, with not too much knowledge, but the ability to think. But then when you find out something new, you always deal with novel things. So that's inevitably more thinking based, than knowledge based. I like these who think about theory, and not just put together some things in the hope that they may start to work.
Title: Re: A possible violation of the Law of Conservation of Energy
Post by: ayeaye on May 16, 2015, 08:58:22 PM
I mean, when the moving magnet is small, then it can be considered that it doesn't change the magnetic field of the other magnets significantly, and all my drawings may remain true. When the magnet is stronger, it affects the magnetic field of the other magnets somewhat, but asymmetry should still remain.

Asymmetry, inducing and canceling the field, especially the magnetic field and the electrostatic field, these are the irregularities that may provide overunity. And these fields are everywhere, in magnets, coils, elasticity of objects, between molecules in the liquid.

Of the overunity experiments, what is known to work is the Tesla's radiant energy receiver. This qualifies as overunity, in that the energy comes from an unknown source. But it doesn't show the cause of overunity. It is also not clear whether this device really receives anything, or are these narrow pulses generated in that same sheet metal. So the experiments with magnets using asymmetry of their fields, are likely the simplest experiments that also show the reason of overunity.